You are on page 1of 141

Barron'sEducational Sertes, Inc. $6.

95

LOGCAND

I " hu+dities and in the social and management sciences. "1 *

Presupposes
only some
high school
algebra
0
Provides
slow
cultivation
of manipulative
skills
LGE

WOODBURY, NEW YORK


All rights reserved.
No part of this book may be reproduced
in any form. by photostat, microfilm, xerography, or
any other means, or incorporated into any information
retrieval system, electronic o r mechanical, without
the written permission of the copyright owner.

All inquiries should beaddressed to:


Barron's Educational Series, Inc.
1 I3 Crossways Park Drive
Woodbury, New York 11797

Librory of Congress Catalog No. 75-1006


International Standard Book No. 0-81 20-0537-6

t i z aj-e~wrSJs
~ ~ (la
Lcvitz, Kathleen.
Logic and Boolean algebra.
Bibliography:
Includes tndex.
1. Logic, Symbolic and mathematical. 2, Algebra,
-.,. .." -.., - --
Boolean. I. Levitz, Hilbert, joint author. 11 T~tle.
,.,.,*
TO O U R M E N T O R S

JOE L. MOTT
KURT SCHUTTE
Contents
PREFACE vii
INTRODUCTION viii

1 sentence Composition 1
1.1 The Basic iogical Operations 1
1.2 Truth Values 2
1.3 Alternative Translations 7
1.4 Converses and Contrapositives 8
1.5 Logic Forms and Truth Tables 9
1.6 Tautologies, Contradictions, and Contingencies 14
1.7 Sentential Inconsistency 15
1.8 Constructing Logic Forms from Truth Tables 16

2 Algebra of Logic 21
2.1 Logical Equivalence 21
2.2 Basic Equivalences 23
2.3 Algebraic Manipulation 24
2.4 Conjunctive Normal Form 29
2.5 Reduction to Conjunctive Normal Form 30
2.6 Uses of Conjunctive Normal Form 33
2.7 Disjunctive Normal Form 35
2.8 Uses 6f Disjunctive Normal Form 38
2.9 Interdependence of the Basic Logical Operations 40

3 Analysis of Inferences 45
3.1 Sentential Validity 45
3.2 Basic Inferences 49
3.3 Checking Sentential Validity of Inferences by R e
peated Use of Previously Proven Inferences 50

4 Switching C h i t s 53
4.1 Representing Switching Circuits by Logic Forms 53
4.2 Simplifying Switching Circuits 58
Preface

T h i s book was intended for students who plan to study in the hu-
manities and in the social and management sciences. Students in-
terested in the physical and natural sciences, however, might also
find its study rewarding. All that is presupposed is some high school
algebra. The authors strongly urge that the topics be studied in the
order in which they appear and that no topics be skipped.
In recent years there has been considerable divergence of opinion
among mathematics teachers as to the degree of abstraction, rigor,
formality, and generality that is appropriate for elementary courses.
We believe that the trend lately has been to go too far in these direc-
tions. Quite naturally, this book reflects our views on this question.
Although the subject matter is considered from a modern point of
view, we have consciously tried to emulate the informal and lucid
style of the better writers of a generation ago. Manipulative skills
are cultivated slowly, and the progression from the concrete to the
abstract is very gradual.
We wish to extend our thanks to our typist Susan Schreck and to
Matthew Marlin and Anne Park, who were students in a course from
which this book evolved. We also wish to thank the editorial con-
sultants of Barron's for their helpful suggestions.

Tallahassee, Florida
Introduction

L o g i c is concerned with reasoning. Its central concern is to dis-


tinguish good arguments from poor ones. One of the first persons to
set down some rules of reasoning was Aristotle, the esteemed philoso-
pher of ancient Greece. For almost two thousand years logic re-
mained basically as Aristotle had left it. Students were required to
memorize and recite his rules, and generally they accepted these rules
on his authority.
At the end of the eighteenth century Kant, one of the great philoso-
phers of modern times, expressed the opinion that logic was a com-
pleted subject. Just fifty years later, however, new insights and results
on logic started to come forth as a result of the investigations of
George Boole and others. In his work, Boole employed symbolism in
a manner suggestive of the symbolic manipulations in algebra. Since
then, logic and mathematics have interacted to the point that it no
longer seems possible to draw a boundary line between the two.
During the last forty years some deep and astounding results about
logic have been discovered by the logician Kurt GBdel and others.
Unfortunately, these results are too advanced to be presented in this
book. We hope that what you learn here will stimulate you to study
these exciting results later.
Finally, we must tell you that complete agreement does not yet
exist on the question of what constitutes correct reasoning. Even in
mathematics, where logic plays a fundamental role, some thoughtful
people disagree on the correctness of certain types of argumentation.
Perhaps someday, someone will settle these disagreements once and
for all.

viii
Sentence Composition
1.1
The Basic Logical Operations
Compound sentences are often formed from simpler sentences by
means of the five basic logical operations. These operations and their
symbols are:
conjunction A
disjunction v
negation 1
implication -,
bi-implication -
The symbols are usually read as follows:

SYMBOL TRANSLATION
A and

I 1 II not I
I 11 ... I
I -
-,
1
if then...
if and only if I
Note that a good way to keep from confusing the symbols A and v is to
remember that A looks like the first letter of the word "AND."
If the letters A and B denote particular sentences, you can use the
logical operations to form these compound sentences:

A A B , read AandB
A v B, read A or B
iA, read it is not the case that A
A -+ B, read if Athen B
A - B, read AifandonlyifB
2 SENTENCE COMPOSITION

Examples. Let A be the sentence: "Snow is white."


Let B be the sentence: "Grass is green."
Then A A B is the sentence: "Snow is white and grass is
green."
Let A be the sentence: "Humpty Dumpty is an egg."
Then 1 A is the sentence: "It is not the case that
Humpty Dumpty is an egg."

Let A be: "Jack is a boy."


Let B be: "Jill is a girl."
Then A -+B is: "If Jack is a boy, then Jill is a girl."
Let A be: "Birds By."
Let B be: "Bees sting."
Let C be: "Bells ring."
Then -IA -,(B v C) is: "If it is not the case that birds
fly, then bees sting or bells ring."

Note that in the last example, the "not" sign applies only to the sen-
tence A. If we had wanted to negate the entire sentence A
-
-+(B v C),
we would have enclosed it in parentheses and written 1 [A (B v C ) ] .
Then it would read, "It is not the case that if birds fly, then bees sting
or bells ring."

A property of declarative sentences is that they are true or false, but


not both. If a sentence is true, it has truth value t. If it is false, it has
truth value f. You can compute the truth value of a compound sentence
built from simpler sentences and logical operations if you know the
truth values of the simpler sentences. This can be done by means of
tables. The table for the conjunction operation is given below. Here is
how to read it. On a given row, the extreme right-hand entry shows
the truth value that the compound sentence A A B should have if the
sentences A and B have the truth values entered for them in that row.
1.2 TRUTH VALUES 3

TABLE FOR A ("AND")

Examples. Use the table just given t o find the truth values of these
compound sentences:
a ) Giants are small and New York is large.
b) New York is large and giants are small.
c ) America is large and Russia is large.

ANSWERS: a ) First label each part of the sentence with its own
truth value.

f t
T_-A----7
[Glants are small] A
&
[New York is large].

Now look in the table t o see which row has the values
f, t (in that order) entered in the two left-hand col-
umns. This turns out t o be the third row (below the
headings). Looking to the extreme right of that row,
you will find that the entire sentence has the truth
value f.

b) Labeling each part with its truth value, we get

Now look in the table t o see which row has the values
t, f (in that order) entered in the two left-hand col-
umns. This turns out t o be the second row. Looking
t o the extreme right of that row, you will find that
the entire sentence has the truth value f.
4 SENTENCE COMPOSITION

C) First label the parts:


t
[America is largJ A
A
[Russ~a1s large].
The first row of the table indicates that the entire
sentence has the truth value t.
We make the tables for the other four logical operations in a similar
way:
TABLE FOR TABLE FOR
v ( 4 6 0 ~ ) -I ("NOT")

TABLE FOR
("IF ...
THEN
-,
...")
TABLE FOR -
("IF AND ONLY IF ")

According to the table for v, the disjunction A v B is a true sen-


tence if A is true, if B is true, or if both A and B are true. Unfor-
tunately, in ordinary conversation people do not always use "or" this
way. However, in mathematics and science (and in this book), the
sentence A v B is considered true even in the case where A and B are
both true.
The implication operation presents a similar problem. Quite often
"if A, then B" indicates a cause-and-effect relationship as in the sen-
tence:
If it rains, thegame will have to bepostponed.

and-effect relationship in affirming the truth of A -


Mathematicians and scientists, however, do not require such a cause-
B, and our
1.2 TRUTH VALUES 5

table has been set up in accordance with their time-honored conven-


tions.

Examples. Use the tabbs just given t o compute the truth values of
the f o l l o & g sentences:
a ) A W equals six, then three equals three.
b) (5 = 7) v (6 = 8).

-
-
c ) It is not the case that three equals three.
d) [(2 = 4) v (3 = 3)] [(5 = 0) A (6 = I)].

ANSWERS: First label the parts with their truth values:


f t
&
a) [two equals six] + [three equals three]
The third row of the table for + shows that the
entire sentence gets the truth value t.

The fourth row of the table for v shows that the en-
tire sentence gets the truth value f.

i[ihree equals three]

The first row of the table for 1 shows that the en-
tire sentence gets the truth value f.
d) This one will involve the use of three tables because
it contains three logical operation symbols.
First label the elementary parts:

From the third row of the table for v you can see
that the part t o the left of the arrow gets the truth
value t. From the fourth row of the table for A , you
6 SENTENCE COMPOSITION

can see that the part to the right of the arrow gets
the truth value f. Now labeling the parts to the left
and right of the arrow with the truth values just com-
puted for them, you have
f

\
From the second row of the table for you can
+

see that the entire sentence should have the truth


value f.

Exercises 1.2
1. Let A denote "Geeks are foobles" and let B denote "Dobbies are
tootles." Write the English sentences corresponding to the following:

2. Let A denote "Linus is a dog," let B denote "Linus barks," and let
C denote "Linus has four legs." Write each of the following sentences
in symbolic form:
(a) Linus is a dog and Linus barks.
(b) Linus is a dog if and only if Linus barks.
(c) If it is not the case that Linus is a dog, then Linus barks.
(d) If Linus is a dog, then (Linus barks o r Linus has four legs).
(e) If (Linus barks and Linus has four legs), then Linus is a dog.
(f) (It is not the case that Linus barks) if and only if Linus is a dog.
(g) It is not the case that (Linus is a dog if and only if Linus barks).
-
3. Let A denote " 1 + 1 = 2" and let B denote "2 2 = 2." Use the
tables to find the truth values of the following sentences:
(a) A v B (h) B-.A -
(b) A v 1B (i) 1A + ( ~ B AA)
(c) 1A v B (j) iA - B
(d) 1A A B (k) 1B--tA
(e) A A -I B (1) ( 1 A v B) ~ ( A B)
A
(f) 1A-B (m) (B v A) v i (B v A)
(g) lA--1B (n) (B+ A)-(A* B)
t . a

t I
1.3 ALTERNATIVE TRANSLATIONS 7

4. Let A and B be sentences. Assuming that B has truth value f, use


the tables to find those truth values for A which make the following
sentences true,
(a) 5 -+A
(b) A A B
(c) 1A-B
(d) (A v B) +B
(e) ( 1 B - A)- A

1.3
Alternative Translations
In English there are many ways of saying the same thing. Here is a list
of some of the alternative ways which can be used to translate the
logical operation symbols.
A and B Not only A but B
A but B A although B
Both A and B
AorB Either A or B
A or B or both A and/or B
[found in legal
documents1
A doesn't hold
It i s not the
case that A
If A, then B. A is a sufficient
condition for B
A only if B B is a necessary
condition for A
A implies B
B provided
that A
A-B A if and only
if B
A exactly
when B
8 SENTENCE COMPOSITION

Exercises 1.3
1. Let A be: peter is a canary.
Let B be: Joe is a parakeet.
Let C be: Peter sings.
Let D be: Joe sings.
Translate the following into symbols.
(a) Joe is a parakeet and Peter is a canary. =- 3-

(b) Although Joe does not sing, Peter sings.


(c) Peter sings if and only if Joe does not sing.
(d) Either Peter is a canary or Joe is a parakeet.
(e) Peter sings only if Joe sings.

2. Let M be: Mickey is a rodent.


Let J be: Jerry is a rodent.
Let T be: Tom is a cat.
Translate the following into symbols.
(a) Although Mickey is a rodent, Jerry is a rodent also.
(b) Mickey and Jerry are rodents, but Tom is a cat.
(c) If either Mickey or Jerry are rodents, then Tom is a cat.
(d) Jerry is a rodent provided that Mickey is.
(e) Either Mickey isn't a rodent or Jerry is a rodent.
(f ) Jerry is a rodent only if Mickey is.
(g) Tom is a cat only if Jerry isn't a rodent.

@
onverses and Contrapositives
Suppose you are given an implication A + B. Two related implica-
tions are given special names.

B -+ A is called the converse


.- of A + B.
1 B 4 iA is called the contrapositive of A + B.

The truth value of an implication and its converse may or may not
agree. Below are given some examples to show this. Later you will see
that an implication and its contrapositive always have the same truth
value.
1.5 LOGIC FORMS 9

Examples. Let A be: 1 = 2


Let B be: 2 is an even number.
Then A -, B has truth value t, while its converse B + A
has truth value f.
Let E be: 2 is an even number. '
Let 0 be: 3 is an odd number.
Then E -,0 has truth value t, and its converse 0
also has truth value t.
- E

Exercises 1.4
1. Let D be: Ollie is a dragon.
Let T be: Ollie is toothless.
Let R be: Ollie roars.
Represent each of the following sentences symbolically. Then write
the 9nverse and the con!r_a~ositive.of each sentence, both in symbols
and in English.
(a) If Ollie is a dragon, then Ollie roars.
(b) If Ollie is toothless, then Ollie does not roar.
(c) Ollie roars only if Ollie is a dragon.

2. Let A and B be two sentences. If A has truth value t, which truth

(a) A -
value must B have to insure that:
B has truth value t?
(b) the contrapositive of A -B has truth value t?
(c) the converse of A -. B has truth value t?
3. Give examples of implications that have truth value t such that:
(a) the converse has truth value t.
(b) the converse has truth value f.

1.5
Logic Forms and Truth Tables
Logical symbolism enables us to see at a glance how compound sen-
tences are built from simpler sentences. Meaningful expressions built
from variable symbols, logical operation symbols, and parentheses
are called logic forms. Capital letters like A, B, C . . . can be used as
10 SENTENCE COMPOSITION

the variable,symbols in constructing logic forms. The following are


examples of logic forms:
A-B
B
AvC
(iA-B)-.B

Note that a single variable symbol is acceptable as a logic form.


Logic forms will often be discussed. When talkingabout logic forms,
heavy-type capital leters like A, B, C (with or without numerical sub-
scripts) will be used to denote them. Thus in a given discussion, you
might use the symbol B to denote the logic form

Remember:
1 . Heavy type capital letters, like A, denote entire logic forms.
2. Regular capital letters, like A, are variable symbols which appear
in a logic form.
You have already seen that the truth value of a logic form is deter-
mined by the truth values assigned to its variable symbols. Let A and
B be variable symbols. A preceding section contained tables listing the
values assigned to each of the logic forms:

i A AvB A A B A-B AuB

for given truth values of A and B. The tables are examples of truth
tables. Using these five tables, you can construct the truth table of
any logic form. As an example, consider the truth table of the logic
form

(A v C) -,(B -+ A).

In each row give an assignment of truth values to the variable symbols


A, B, and C, and at the right-hand end of the row list the value of the
entire logic form for that assignment. The optional columns headed
"A v C" and "B -,A" are included to help fill in the table. If you
feel sufficiently confident, you can omit such intermediate columns
when building future truth tables.
1.5 LOGIC FORMS I1

TRUTH TABLE FOR (A v C) -- (B7A)

+
optional columns
We suggest that you first fill in the columns under the variable
symbols A, B, C . . . and then complete these columns one at a time.
Below are repeated the first three columns of the preceding table.
Note that the places in the columns which have t as entries are shaded,
and observe that there is a pattern to this shading. By following the
pattern you can be sure that a logic form with n distinct variable
symbols will have a truth table with 2" rows and that these rows dis-
play aN the possible ways of assigning truth values to the variable
symbols. Note that the top half of the left-hand column consists only
o f t entries, the bottom half of f entries. In the next column blocks
o f t entries and blocks of f entries are alternated; each block consists
of 114 the total number of rows in the table. In the third column, each
block makes up 118 of the total number of rows, etc.
12 SENTENCE COMPOSITION

Exercises 1.5
1. Construct the truth table for each of the following logic forms:
(a) A + 1A (b) (C A 1 B) v (A B)

-
-+

(c) ~ ( A v C ) (d) [ ( A ~ B ) v ( C + ~ D ) ] - + ( A A ~ D )
(e) [(B v A) A C] [(B A C) v (A A C)]

2. Use truth tables to determine the truth value of D, given the fol-
lowing:
(a) C is true and C A D is true
(b) C is false and D v C is true
(c) 1 D v 1 Cisfalse
(d) C is false and (C A D) -+ (C v D) is true

3. a) Write a logic form corresponding to the following sentence:


If the stock's value rises or a dividend is declared, then the stock-
holders will meet if and only if the board of directors summons
them, but the chairman of the board does not resign.
b) Determine the truth value of the preceding statement under the
following assumptions by (partially) filling out the truth table for its
logic form.
i) The stock's valuerises, no dividend is declared, the stockholderswill
meet, the board of directors summons the stockholders, and the chair-
man resigns.
ii) The stock's value rises and a dividend is declared, the board of
directors fails to summon the stockholders, and the chairman resigns.

4. Horace, Gladstone, and Klunker are suspected of embezzling com-


pany funds. They are questioned by the police and testify as follows:
Horace: Gladstone is guilty and Klunker is innocent.
Gladstone: If Horace is guilty, then so is Klunker.
Klunker: I'm innocent, but at least one of the others is guilty.

(a) Assuming everyone is innocent, who lied?


(b) Assuming everyone told the truth, who is innocent and who is
guilty?
(c) Assuming that the innocent told the truth and the guilty lied,
who is innocent and who is guilty?
7

1.5 LOGIC FORMS 13

[Hint: Let H denote: Horace is innocent.


Let G denote: Gladstone is innocent.
Let K denote: Klunker is jnnocent.
Now symbolize all three testimonies and make one single truth table
with a column for each testimony. The desired information can be
read from the table.]
5. Agent 006 knows that exactly one of four diplomats is really a spy.
He interrogates them, and they make the following statements:
Diplomat A: Diplomat C is the spy.
Diplomat B: I am not a spy.
Diplomat C: Diplomat A's statement is false.
Diplomat D: Diplomat A is the spy.
(a) If 006 knows that exactly one diplomat is telling the truth, who
is the spy?
(b) If 006 knows that exactly one diplomat's statement is false,
who is the spy?
[Hint: Let A denote: Diplomat A is a spy.
Let B denote: Diplomat B is a spy.
Let C denote: Diplomat C is a spy.
Let D denote: Diplomat D is a spy.
Now symbolize the reply of each diplomat and make a single truth
table with a column for each reply. The desired information can be
read from the table.]
*6. A certain college offered exactly four languages: French, German,
Russian, and Latin. The registrar was instructed to enroll each stu-
dent for exactly two languages. After registration, the following facts
were compiled:
i) All students who registered for French also registered for ex-
actly one of the other three languages.
ii) All students who registered for neither Latin nor German regis-
tered for French.
iii) All students who did not register for Russian registered for
at least two of the other three languages.
iv) No candidate who registered for Latin and German registered
for Russian.
Did the registrar actually follow his instructions?

*This denotes a difficult problem


14 SENTENCE COMPOSITION

[Hint: Let x be an arbitrary student at the college.


Let F denote: x registered for French.
Let G denote: x registered for German.
Let L denote: x registered for Latin.
Let R denote: x registered for Russian.
Symbolize each of the four facts. Make a truth table with a column
for each of the four facts. Locate the rows for which all four facts
have truth value t. Examine these rows closely.]

1.6
Tautologies, Contradictions,
and Contingencies
Certain logic forms have truth tables in which the right-hand column
consists solely of t's. These forms are called tautologies. Hence a
tautology is a logic form which has truth value t no matter what
values are given to its variable symbols. If the right hand column of
the truth table of a logic form consists solely off's, the logic form is
called a contradiction. Thus a contradiction has truth value f no mat-
ter what values are given to its variable symbols. If a logic form is not
a tautology and not a contradiction, it is called a contingency.

Example 1. Tautology: A v 1A
Contradiction: A A iA
Contingency: A B -+

Notice that if the variable symbols of a tautology are replaced by


sentences, the compound sentence is always true.

Example 2. You have already observed that A v iA is a tautology.


Thus the sentence:
A 1A
A

'it is raininiorct is not raining\


is a true sentence.
The truth of the sentence in this example does not
depend on any facts from meteorology. It is true by
1.7 SENTENTIAL INCONSISTENCY IS

virtue of the way it is built up from its parts by means


of the logical operations. In other words, it is true by
virtue of its form. Logicians call a sentence like this
one a substitutive instance of a tautology.

Exercises 1.6
1. Make truth tables for the following logic forms. Indicate which
are tautologies, which are contradictions, and which are contingencies.
(a) A -+A (b) A -- (B A)
-+

(c) (A v B) -,(A A 1C) (d) A v ( 7 A A C)


(e) A-+(TAAB) (f) 7 (A A B ) - ( ~ A v l B)

1.7
Sentential Inconsistency
Let A , , A , . . . A, be a collection of logic forms. These logic forms are
said to be sententially inconsistent if their conjunction is a contradic-
tion. Otherwise, they are said to be sententially consistent.
To test a collection of logic forms A,, A,. . . A, for sentential inconsis-
tency, you have only to form their conjunction A, A A, A -. A A,,
make a truth table for this conjunction, and look to see whether the
right-hand column of the table has all f's. If this is so, then they are
sententially inconsistent. If there is at least one t, then they are sen-
tentially consistent.
If, after symbolizing a collection of sentences, you find that the re-
sulting collection of logic forms is sententially inconsistent, then you
would know that these sentences are built from their elementary sen-
tences in such a way that it is impossible for all of them to be true.

Exercises 1.7
1. Test the following collection of logic forms for sentential incon-
sistency:
16 SENTENCE COMPOSITION

2. Symbolizethe following sentences and test the resulting logic forms


for sentential inconsistency.
(a) Either a recession will occur or, if unemployment does not de-
crease, wage controls will be imposed. If a recession does not oc-
cur, unemployment will decrease. If wage controls are imposed,
unemployment will not decrease.
(b) If imports increase or exports decrease, either tariffs are im-
posed or devaluation occurs. Tariffs are imposed when and only
when imports increase and devaluation does not occur. If exports
decrease, then tariffs are not imposed or imports do not increase.
Either devaluation does not occur, or tariffs are imposed and ex-
ports decrease.
3. After discovering his immense popularity with the American
people, Bagel the beagle decided to run for president. He called to-
gether his top political advisors for a brainstorming session. Out of
this session came the following advice:
Owl: Beagles can't be president, or the country will go to the dogs.
Fox: If a beagle can be president, the country won't go to the dogs.
Bear: Either a beagle can be president, or the country will go the
dogs.
Cai: It's not the case that (the country will go to the dogs and a
beagle can't be president).
Since Bagel's confidence in his advisors was a shade less than ab-
solute, he decided to run a sentential consistency test. What did it re-
veal?

1.8
Constructing Logic Forms from
Truth Tables
You know that for each logic form a truth table can be made. In this
section you will learn how to reverse the procedure. Here we have a
truth table for an unknown logic form with n given variable symbols.
The problem will be to construct a logic form having this given table
for its truth table.
Given the truth table, the following procedure is employed to con-
struct the desired logic form.
1. For each row whose right hand entry is t, make a check m a r k d
to the right of that row.
1.8 CONSTRUCTING LOGIC FORMS FROM TRUTH TABLES 17

2. To the right of each check mark */ write a sequence of n terms


(one corresponding to each variable symbol) as follows: if in that
row a variable symbol has the value t entered for it, then the cor-
responding term is to be that variable symbol itself; if the variable
symbol has the value f entered for it, then the corresponding term is
to be the negation of that variable symbol.
3. To the right of each sequence of terms which you made in step 2,
write the conjunction of those terms.
4. Beneath the table, form the disjunction of all the conjunctions
which you made in step 3.
The disjunction formed in step 4 will be the desired answer.

Example. Construct a logic form having the following truth table.

t f f

ANSWER: Step 1 I Step 2 1 Step 3


18 SENTENCE COMPOSITION

The result of Step 4 is the desired logic form. To check


your work, make the truth table for the logic form just
constructed.

Comparing this table with the table you were given, you
can see that they coincide. Hence
( A A B AC~) v ( A A l B A C ) Y ( T AA ~ B A C )
is a correct answer.

Probably you have noticed that this method doesn't mention what
to do in case the right-hand column of the given table only has f's.
This case is even easier; just make a contradiction of the form
(X A 7 X) for each variable symbol X,and then take the disjunction of
these contradictions.

Exercises 1.8
1. Using the procedure outlined in this section, find a logic form for
each of the following truth tables:
1.8 CONSTRUCTING LOGIC FORMS FROM TRUTH TABLES 19

Check each answer by making the truth table of each logic form con-
structed.

2. Find a logic form with the 3 variable symbols A, B, C, which has


the following truth table:

'3. The people who live on the banks of the Ooga River always tell
the truth or always tell lies, and they respond to questions only with a
yes or a no. An explorer comes to a fork in the river, where one branch
leads to the top of a mile high waterfall and the other branch leads to
a settlement. Of course there is no sign telling which branch leads to
the settlement, but there is a native, Mr. Blanco, standing on the

*This denotes a difficult problem.


tO SENTENCE COMPOSITION

shore. What yes-or-no question should the explorer ask Mr. Blanco to
determine which branch leads to the settlement?
[Hint: Let A stand for "Mr. Blanco tells the truth," and let B stand
for "The left-hand branch leads to the settlement." Construct, by
means of a suitable truth table, a logic form involving A and B such
that Blanco's answer to the question will be "yes" (i.e., true) if and
only if B is true.]
Algebra of Logic

-
Let A and B be logic forms. A and B are logically equivalent if and
only if the logic form A B is a tautology. If A is logically equivalent
to B, it is indicated by writing A = B.
Suppose A i B. Then if all the variable symbols appearing in
A or B are listed, the assignments of truth values to these symbols
which make A true will be precisely those which make B true.

Example I. Prove: (A -. B) = 1 (A A 1 B)

TO do this, construct the truth table of the logic form


-
ANSWER:

(A + B) 1 (A A 1 B)

which, for brevity, will be denoted by E.

From the truth table you can see that E is a tautology.


Hence:
A - - B = ~ ( A A ~ B )

Example 2. Prove: A -+ (B -+ A) C v -I C

Construct the truth table for the logic form


-
ANSWER:

[A -,(B -. A)] (C v -I C)

which is denoted by E.
22 ALGEBRA OF LOGIC

A I B I C I A-(B-A) I CvlC I E
t t t t t t
t t f t t t
t f t t t t

t I t
f l t l f I t t I t
f i t t t t
f f f t t t
By inspecting the truth table for E, you can conclude:

A-(B-A) = (C v i C)

The second example shows that the tautologies A -,(B A) and -+

C v iC are logically equivalent. Actually it is true that all tautologies


are logically equivalent, and all contradictions are logically equiva-
lent.

Exercises 2.1
1. By constructing the appropriate truth table, prove or disprove the
following claims:

(a) ( A v B ) -= l ( i A A 1 B )
(b) ( A v B ) o l ( l B ~ i C )
(c) =
A A (B v C) (A A B) v (A A C)
(d)
(e)
C A (B v D) = (C v D) A (B v D)
A -,(B -+C) = (A -. B) C -
2. Construct a logic form distinct from, but logically equivalent to,
the logic form

A -+ (B - C).

[Hint: Make the truth table for A (B -. C). Then use the pro=-
-+

dure for constructing a logic form, given its truth table. Check that
the logic form so obtained is logically equivalent to A (B -+ C).] -
2.2 BASIC EQUIVALENCES 23

Basic Equivalences
Here is a list of twenty basic equivalences which will be used repeatedly
throughout this book. You should familiarize yourself with all of the
equivalences listed. They can be easily verified by the truth table
technique which you used earlier to check for logical equivalency.
(In fact, you already verified a couple of them in previous exercises.)
The symbol T is used to denote the particular tautology A v 1 A,
and the symbol F is used to denote the particular contradiction

Basic Equivdences
I. i i A = A law of double negation
2. A V B Z B V A
commutative laws
3. A A B P B A A

associative laws

6 . A v (B A C) = (A v B) A (A v C)
distributive laws
7., A A (B v C) = (A A B) v (A A C)

de Morgan's laws

idempotent laws
-

absorption laws

identity laws

domination laws

18. A - B = l A v B arrow law

120. A -
19. A - , B = i B - i A
B = (A -, B) A (B + A)
law of contraposition
double arrow law
24 ALGEBRA OF LOGIC

We close this section with a few observations. In Chapter 1 we


introduced the concept of "contrapositive" of an implication. Basic
equivalence 19 asserts that an implication is logically equivalent to its
contrapositive.
From your basic algebra course you may be familiar with the
commutative, associative, and distributive laws. In contrast to the
situation in ordinary algebra, there are two distributive laws here.
Note that one of these may be obtained from the other by interchang-
ing the A and v symbols. In ordinary algebra this doesn't work. If you
interchangethe + and . symbols in the distributivelaw
a.(b + c) = (a-b) + (a-c)
you get

which is false in ordinary algebra. (Note, for example, that it fails


when a, b, c, each have the value 1.)

2.3
Algebraic Manipulation
In this section you will learn another method for proving that two
logic forms are logically equivalent. For this procedure you need the
following two rules.
I. Replacement rule. If part of a logic form A is replaced at one or
more occurrences by a logically equivalent form, then the result is
logically equivalent to the original form A.
11. Transitivity rule. If A, B, and C are logic forms such that
A = BandB = C,thenA s C.
The technique used to show logical equivalency here resembles the
process used in algebra to ;how that two algebraic expressions are
equal. Starting with one of the logic forms and making successive
applications of the replacement rule and the basic equivalences of
the preceding section you convert it into the other logic form. Then,
2.3 ALGEBRAIC MANIPULATION 25

by using the transitivity rule, you can conclude that the two logi_c
forms are logically equivalent.
Here are a few examples to show the algebraic manipulation of
logic forms. Study these examples carefully before attempting the ex-
ercises which follow.

Example 1. Prove: A A (B v -I A) A A B

PROOF: Start with the left side and, in a series of steps (listed
vertically), manipulate your way to the right side:
A A ( B v 1 A)
distributive law
(A A B) v (A A 1 A)
definition of F
(A A B) v F
identity law
A A B
Since you went from the left side to the right side in steps
which leave each line logically equivalent to the one pre-
ceding it, you can conclude by the transitivity rule that
the desired equivalence holds.

Example 2. Show that 1(A -, B) = A A 1B

ANSWER: 1 (A -+ B)
arrow law
1( iAvBj
de Morgan's law
1 1 A A l B
double negation law
A A ~ B

Since you started with the left side of the equivalence in


qtlestion and ended with the right side, you can conclude
by the transitivity rule that the equivalence holds.
Exercises 2.3
1. Establish the following equivalences using algebraic manipulations.
(a) AvB= lA--B
(b) l(A--B)=AAlB
(c) A -+(B -- C) = (A A B) -- C
(d) (A A B) v ( 7 A A C) = (A -- B) A [C v (A A B)]
2. Simplify the following expressions. (Try to minimize the total
number of occurrences of operation symbols and variable symbols.
Answers won't be unique.)
1A v ( A v B )
i (7 B v A )
1A A ( A v B )
(A A B) V (A A 1B)
1 (A A 1 (A v B))
A A(A v B v C )
[(lBvA) AA]-A
(A v B) A [(B v A) AC]
1 (A v 1 (B A C))
A A ( 1 A vB)
A v [ q A v ( B ~ C)] l
[ ( i A A 1 B ) v i ( A v B ) ] - + ( - I A A 1B)
[A A (B A C)] v [ 1 A A (B A C)]
E A D A [ i [(A-B)-(1 B-. -I A ) ] - + C ]

3. Negateeach of the following sentences, and then express the an-


swer in such a way that only elementary (not compound) parts of the
sentence are negated.
(a) Freedom of the press is an important safeguard of liberty, and
in protecting it, our courts have played a major role.
(b) If a politician seeks the presidency, and he has sufficient fi-
nancial backing, then he can afford to appear on nationwide televi-
sion frequently.
(c) A man has self respect if he is contributing to a better society.
(d) We can halt pollution only if we act now.
(e) If a man cannot join the union, he cannot find a job in that
factory and he must relocate his family.
(f ) If a worker can share on the company profits, he works harder
and he demands fewer fringe benefits.
2S ALGEBRA OF LOGE

5. Show that you can manipulate from one side of the absorption law
to the other side, using only the remaining 18 basic equivalences.
6. The Internal Revenue Service lists the following three rules as
guidelines for filing tax returns:
(a) A person pays taxes only if he is over 18 years of age, or has
earned $1000 during the past twelve months, or both.
(b) N o widow must pay taxes unless she has earned $1000 during
the past twelve months.
(c) Anyone over 18 years of age who has not earned $1000 during
the past twelve months does not pay any taxes.
Find a simpler form for these rules.
[Hint: Let x be an arbitrary person.
Let T denote: x is a taxpayer.
Let W denote: x is a widow.
Let A denote: x is over 18 years of age.
Let E denote: x earned over $1000 in the past twelve months.
Then the guidelines given can be represented symbolically by:

Simplify this logic form.]

*7. In the Tooba tribe, the elders use the following rules to decide a
man's role in the tribal society. Try to replace this set of rules by a
simpler (but logically equivalent) set of rules.
(a) The warriors shall be chosen from among the hunters.
(b) Any Tooba man who is both a farmer and a hunter should
also be a warrior.
(c) No farmer shall be a warrior.
[Hint: Let x be an arbitrary Tooba man.
Let W denote: x is a warrior.
Let F denote: x is a farmer
Let H denote: xis a hunter.]

*This denotes a difficult problem.


2.4 CONJUNCTIVE NORMAL FORM 29

2.4
Conjunctive Normal Form
Logic form A is a conjunctive normal form if it satisfies one of the fol-
lowing conditions:
i) A is a variable symbol.
ii) A is the negation of a variable symbol.
iii) A is a disjunction of two or more terms, each of which is either a
variable symbol or the negation of a variable symbol.
iv) A is a conjunction of two or more terms, each of which is one of
the three types listed above.
It is very important to be able to recognize a conjunctive normal form.

Example 1. The following logic forms are conjunctive forms:


A (satisfies condition i)
1 B (satisfies condition ii)
TCvBvA (satisfies condition iii)
( 7 C v B v A) A A A 1 B (satisfies condition iv)
( iA v B) A (C v A) A (B v C) (satisfies condition iv)

The following logic forms are not conjunctive normal


forms:
A+B
( A A B)v i C
(AvB)rr(iB4A)
7 (A * B)

To help fix the idea in your mind, think of a conjunctive normal


\
form as a logic form which looks like

where the heavy-type letters denote logic forms no more complicated


than single variable symbols or negations of variable symbols. We
are allowing as special cases of this, the case where only one of the
bracketed expressions actually appears, and the case where some of
the bracketed expressions have only one term. The bracketed expres-
sions between the conjunction symbols are called the cwjunets of
38 ALGEBRA OF LOGIC

the normal form. If the conjunctive normal form has no conjunction


symbols, the entire expression is thought of as a single conjunct. Thus,
A v B v -IC has only one conjunct, namely the expression
A v B vlC.

Exercises 2.4
1. Determine which (if any) of the following logic forms are conjunc-
tive normal forms.
(A v B v 1B) A ( A v C)
(AvBv ~B)A(AAC)
(A + B) v (C A 1 B)
AAB
~ A A B
1[(A v B) A (A v C)]
1AA B A l C
A A [B v (C A 1D)]
(A A B) v [(A A C) v (A A iB)]

2.5
Reduction to Conjunctive Normal Form
By using algebraic manipulations it is possible to reduce any logic
form to an equivalent conjunctive normal form.

Example. Find a conjunctive normal form logically equivalent to:

ANSWER: (A A B) V (B A 1C)

distributive law
[(A A B) v B] A [(A A B) v 1 C]
distributive law
[(A v B) A (B v B)] A [(A A B) v 1C]
distributive law
[(A v B) A ( B v B ) ] A [ ( A v 1 C ) A ( B v 1 C)]
associative law
( A v B ) A ( B v B ) A ( A A1 C ) A ( B V 1 C )
2.5 REDUCTION TO CONJUNCTIVE NORMAL FORM 31

The last line is, as desired, a conjunctive normal form.


It is not the simplest answer, but for the intended ap-
plications of conjunctive normal form, this will not
matter.

Note that a logic form might be manipulated into a conjunctive


normal form in various ways, and the answer might come out in
various ways. All answers, of course, would be logically equivalent.
Now it turns out that in reducing to a conjunctive normal form you
need to use just one of the two distributive laws learned in Section 2.2.
If you now switch to an "arithmetical notation" and write

A + B instead of A A B
AB instead of A v B

then you see that of the two distributive laws, the one you need ap-
pears in the new notation as

A(B + C) = AB + AC.
The other distributive law would look rather strange and unfamiliar
in arithmetical notation. It would appear as

A + (BC) = (A + BXA + C).


You are probably so accustomed to working with ordinary numbers
where this second distributive law fails, that we might have confused
you if we had introduced and used arithematical notation earlier in
the book. Fortunately, for reduction to conjunctive normal form you
do not need to make use of this "funny looking" distributive law. So
perhaps in reducing to conjunctive normal form, it might be more con-
venient to use arithmetical notation. For one thing, your expressions
will take up less space. For another, repeated application of the dis-
tributive law to complicated expressions can be handled just as though
you were "multiplying out" an expression of ordinary algebra.
A further space saving device would be to write

A' instead of 1 A.
32 ALGEBRA OF LOGIC

De Morgan's laws would then appear as


(A + B)' = A'B'
(AB)' = A' + B'
Of course, for uncomplicated expressions you may prefer not to use
the new arithmetical notation at all. It is entirely optional.
A good way to reduce to a conjunctive normal form is to perform
the following four steps:
1. Use the arrow laws to remove all -.and -.
2. Use de Morgan's laws repeatedly until the only negated terms are
variable symbols.
3. Switch t o arithmetical notation.
4. Multiply out (as in ordinary algebra).
Naturally, if you get a chance to simplify things along the way by
using the absorption, idempotent, domination, or identity laws, so
much the better.

Example. Find a conjunctive normal form logically equivalent to:


[(A -, B) -,C] v [ D A El

ANSWER: I(A+B)+C]V[DAE]
arrow law
[(~AVB)-,C]V[DAE]
arrow law
[ l (~AvB)vC]V[DAE]
de Morgan's law
[ ( - I7 A A i B) v C] v [ D A El
double negation law
[(A A 1 B) v C] v [D A E]
change notation to arithmetical
[(A + B1)C](D + E)
associative and commutative laws
C(A + B1)(D + E)
multiply out (distributive law)
CAD + CB'D + CAE + CB'E
The last line is the desired conjunctive normal form.

A good way to remember that it is the "and" symbol A which cor-

S m a Public Library
7272 V : -- ~ -
2.6 USES O F CONJUNCTIVE NORMAL FORM 33

responds to the symbol + when reducing to a conjunctive normal


form, is to think of the word CAP, which is made from the first letters
of the words
Conjunctive (normal form) And Plus.

Exercises 2.5
1. Reduce each of the following logic forms to a conjunctive normal
form:

tA-B
i(1BvA)
(A A B) v (A A 1 B)
1 (A A 1 (A v B))
A-B
A A [B v (C A i D)]
(A B) A [ C v (A A B)]
-+

(A A B) V [(A A C) V (A A 1B)]

2.6
Uses of Conjunctive Normal Form
If a logic form A is a conjunctive normal form, there is a way of tell-
ing at a glance whether or not it is a tautology. Just apply the follow-
ing rule:
A conjunctive normal form is a tautology if and only
if in each conjunct there appears some variable symbol
and its negation.
In applying the above rule, do not forget that if no conjunction
symbols actually appear in the conjunctive normal form, the entire
normal form is thought of as a single conjunct.
/-

Example 1. (A v B v i A) A (C v D v E v i D) is a tautology be-


cause of the appearance of A, 1 A in the left conjunct
and D, 1 D in the right conjunct.
( A v B v 1 A ) A ( C V 1 B v D ) A ( E v 1 E) is not a
tautology because the middle conjunct fails to have
some variable symbol and its negation.
34 ALGEBRA OF LOGIC

A v BvDv 1B is a tautology because of the ap-


pearance of B, 1 B.
A vBv i C is not a tautology. Its only conjunct (the
whole expression itself) fails to have some variable
symbol and its negation.

Here is how to test any logic form A for tautology (regardless of


whether A is a conjunctive normal form or not). First reduce A to a
conjunctive normal form. Then you can see at a glance whether the
normal form (and hence A) is a tautology.

Example 2. Is the following logic form a tautology?


A + (B -, A)

ANSWER: Reduce to a conjunctive normal form


A+(B--A) E 1A v(B-+A)
lAv(1BvA)
= 1 A v l B v A
Now 1 A v 1 B v A is a conjunctive normal form,
and it is evident by inspection that it is a tautology.
Consequently, so is A (B -. A).
+

Example 3. Is the following logic form a tautology?

ANSWER: Reduce to a conjunctive normal form

(A-B)-+A = i(i AvB)vA


= ( f l A ~ l B ) v A
= (11 A v A ) A ( ~ B v A )
= (AvA)h(l BvA)
= Arr(1BvA)
Now A A ( 1 B v A) is a conjunctive normal form. By
inspection it is evident that it is not a tautology, so
neither is (A B)-+ A. -+
2.7 DISJUNCTIVE NORMAL FORM 35

In both examples a truth table could have been used just as easily
to settle the question. For expressions with three or more distinct
letters, making a table can be tedious, and the method of conjunctive
normal forms is preferable.
Using a conjunctive normal form to test for tautologies also gives
additional information. If the test shows that a logic form is not a
tautology, the conjunctive normal form helps to find an assignment of
truth values to the variable symbols, which makes the original logic
form have truth value f. A11 that has to be done is to locate a conjunct
which fails to have both a variable symbol and its negation. Then
assign values to the variables so as to make that conjunct false.

Exercises 2.6
1. Test the following to see if they are tautologies using the method
of conjunctive normal form. For each logic form which is not a tau-
tology, use the conjunctive normal form to find an assignment of
truth values to the variables which makes the logic form false.
(a) [A A (A -- B)] -,B
(b) A V 1 A
(c) A -,(A v B)
(d) [(A -. B) A 1A] + B
(el [ 7 (A -, B)1 + (A v B)
(f) ( A v B ] - - A
(g) (A + B) v (C A B)
(h) (A A B) v [(D -. C) v El
( i ) (A++B)A((CVD)--A]

2.7
Disjunctive Normal Form
A logic form A is a disjunctive lorarrl form if its satisfies one of the
following conditions:
i) A is a variable symbol.
ii) A is the negation of a variable symbol.
iii) A is a conjunction of two or more terms each of which is a variable
symbol or the negation of a variable symbol,
iv) A is a disjunction of two or more tirms each of which is one of the
three types described above.
36 ALGEBRA OF LOGIC

Example 1. The following logic forms are disjunctive normal


forms:
A
1 B v C
A A BA ~ C A D
( l B ~ c ) v ( A ~ D )E
v l
The following logic forms are not disjunctive nor-
mal forms:
A-B
( ~ B v C ) A A
(A A C) v (A B) -+

Note that some logic forms, for example the form 1 A, can be
both a conjunctive normal form and a disjunctive normal form.
A disjunctive normal form looks just like a conjunctive normal
form except that the roles of the symbols A and v are reversed. Thus

is a conjunctive form, while


(AA 1 B ) V D

is a disjunctive normal form.


Every logic form A can be reduced to a logic form which is a dis-
junctive normal form logically equivalent to A.

Example 2. Find a disjunctive normal form which is logically equiv-


alent to:
(A + B) + C

ANSWER: (A + B) --+ C
arrow law
i(A- B) v C
arrow law
i(iAvB)vC
de Morgan's law
(AA 1B)vC
The last line is, as desired, a disjunctive normal form.
2.7 DISJUNCTIVE NORMAL FORM 37

Example 3. Find a disjunctive normal form which is logically equiv-


alent to:

ANSWER: A A (B V C) (A A B) V (A A C)

The right-hand side is the answer.

Now it turns out that in reducing to disjunctive normal form you


need to use only one of the two distributive laws. In fact, it's the one
you did not need for reducing to conjunctive normal form. Recall that
in switching to arithmetical notation this distributive law looked un-
familiar. You can avoid dealing with a "funny looking" representa-
tion of this distributive law by reversing the role of the + symbol.
This time write

A + B for AvB
AB for A A B

Again the distributive law that you do need is

A(B + C) = AB + AC.
Any logic form can be reduced to a disjunctive normal form by the
following steps:
1. Eliminate arrows.
2. Repeat the use of de Morgan's laws until negation symbols appear
only in front of variable symbols.
3. Switch to arithmetical notation.
4. Multiply out (like in ordinary algebra).
In order to help you remember that it is the "or" symbol which
should correspond to + in reducing to a disjunctive normal form,
think of the word DOPE whose first three letters come from
Disjunctive (normal form) Or Plus.
Recall that for conjunctive normal form the key word was CAP. Of
course, if the logic form in question isn't very complicated, you may
prefer not to use arithmetical notation.
38 ALGEBRA OF LOGIC

Example 4. Find a disjunctive normal form logically equivalent t o

ANSWER: 1(A A B) A (A ) A C
arrow law
~(AAB)A(~AvC)AC
de Morgan's law
(1Av-i B)A(~AvC)AC
change notation to arithmetical
(A' + B'XA' + C)C
multiply out (distributive law)
A'A'C + B'A'C + A'CC + B'CC.
The last line is a disjunctive normal form.

Exercises 2.7
1. Determine which (if any) of the following logic forms are disjuno
tive normal forms.
(a) A AB (b) A vB
(c) A A (B v C) (d) A v (B A C)
(e) A v 7 CV(AAC) (f) ( A - + B ) v 1C
(B) 1 (A v B) (h) T A A ~ B
2. Reduce each of the following forms to a logic form which is a dis-

-
junctive normal form.
(a) A B
(c) 1 (A v B)
(b) A v B
(dl (A -- B)
(e) A A B A C ( f ) (A v B) A (B -- C)
(g) 17 @ - + C ) l - + A (h) 1 [(B C) A1
-+ -+

( i ) (A v B ) A [(B v A) A C ]

(k) (A -
(j) AA[(~A-B)vC]
B) + [(B C) -- (A -+ C)]
-+

Uses of Disjunctive Normal Form


If a logic form is a disjunctive normal form, there is a way of telling
by inspection whether or not it is a contradiction. Just apply the fol-
lowing rule:
2.8 USES OF DISJUNCTIVE N O R M A L FORM 39

A disjunctive normal form is a contradiction if and


only if in each disjunct there appears some variable
symbol and its negation.

Example 1. D A 1 D is a contradiction.
(A A B) v (A A C) is not a contradiction.
B is not a contradiction.
(A A 1 A) v (A A C) is not a contradiction.
(A A 7 A A B) is a contradiction.
(A A 1 A) v (B A C A 1 B) is a contradiction.
To test an arbitrary logic form to see if it is a contradiction, reduce
it to a disjunctive normal form, and then check the normal form by in-
spection.

Example2. Use reduction to a disjunctive normal form to tell


whether the following logic form is a contradiction.
(A -. 1 B) A (A A B)

ANSWER: ( A - + ~ B ) A ( A A B ) E ( ~ ABV) A~( A A B )


=(~AAAAB)v(~BAAAB)
Each disjunct has some letter and its negation. Therefore, the origi-
nal logic form is a contradiction.
Even if reduction to a disjunctive normal form reveals that a logic
form A is not a contradiction, some useful knowledge can be obtained.
You can use the disjunctive normal form to determine an assignment
of truth values to the variable symbols in A which gives A truth value
t. All you have to do is locate a disjunct which fails to have a variable
symbol and its negation. Then assign values to the variables so as to
make that disjunct true.

Exercises 2.8
1. Reduce each of the following logic forms to a disjunctive normal
form and determine whether it is a contradiction. If it is not a contra-
diction, use the disjunctive normal form to find an assignmint of
a ALGEBRA OF LOGIC

truth values to the variable symbols which makes the original logic
form true.
(a) A+(lA*B)
(b) A+(lAvB)
(c) (A v B) A l ( 1 A - B)
(d) (AA(BvC))A(~A+(~BATC))
2. Use reduction to a conjunctive normal form and to a disjunctive
normal form to determine which of the following logic forms are tau-
tologies, which are contradictions, and which are contingencies.
(a) 1 A v (B -7 C) v (C +A)
(b) [(A + B) + C] -, [(C A)
+ +(D +A)]
(c) (A+B)+[(B-,C)+(A-+C)]
(d) [(A A C) v (B A 1 C)] [ ( l A A C) v ( 1 B
++ A 7 C)]
3. Check the following set of assumptions for sentential consistency.
If the XYZ company builds a new factory or some of its machinery
must be replaced, either the company can obtain a loan or it will be
threatened with bankruptcy. The company can obtain a loan if and
only if it builds a new factory and no machinery must be replaced.
If machinery must be replaced, then the company cannot obtain a
loan or it does not build a new factory. Either the company will be
threatened with bankruptcy, or some of its machiner must be re-
placed and the company can obtain a loan.
[Hint: Symbolize the sentences, take the conjunction and reduce to
a disjunctive normal form. See if it is a contradiction.]

2.9
Interdependence of the Basic
Logical Operations
Previously you were introduced to the five basic logical operations:
A

v
1

.-.
+
2.9 INTERPENDENCE OF LOGICAL OPERATIONS 41

-
The double arrow law mentioned earlier in this chapter shows how to
express the operation in terms of the operations -.and A:
=

--
A-B (A4B)A(B-.A)
It follows that any logic form with occurrences of can be con-
verted to a logic form having no occurrences of by replacing the
left side of the above equivalence with the right side. This shows that
you can get along with just the remaining four logical operations.
Furthermore, the arrow law and de Morgan's law can be used to ex-
press-. and A in terms of v and 1:
A-B=1 AVB

Hence, given any logic form A, you can eliminate all occurrences
of -, -+,and A in favor of v and 1 ,and the new logic form would be
logically equivalency to A.
The above discussion shows that of the five basic operations avail-
able, it is possible to get along with just two of them, namely v and 1 .

Example. Find a logic form logically equivalent to the one below,


but which uses only the connectives v and i:

ANSWER: A ++ B a (A --* B) A (B-+ A)


-- ( ~ A v B ) A ( ~ B v A )
P [ ( ~ A v B ) A lB]v[(iAvB)~Al
-= (i A A 7 B) ~ ( B i A B)
v [ ( l A A A) V ( B A A)]
= ( - ~ A AT B ) v ( B A A )
i(AvB)v i ( i A v iB).

The example above shows one reason why we did not merely in-
troduce the two logical operations v and i at the beginning of the

Many short logic forms, for example A -


book and do all subsequent work using just these two operations.
B, would be expressed by
long unwieldy ones. Thus, even though it is possible to manage in
principle with just two operations, it is convenient to have all five
of them available.
ALGEBRA OF LOGIC

There is a new logical operation in terms of which each of the five


b a s i c operations can be expressed. It is called the Sbeffer stroke, and it
is denoted by a vertical stoke I . Its truth table is:

Since the truth table of A 1 B is the same as the truth table of


i (A A B), we have
A l B =l(A A B).
You have already seen that any logic form can be expressed using
o n l y the operations v and 1.Thus, to conclude that all the basic logi-
cal operations can be expressed in terms of the Sheffer stroke alone,
y o u need only show that v and 1 can each be expressed in terms of I .
Y o u can see that 7 A may be so expressed by observing that:

T h a t A v B can be expressed in terms of the Sheffer stroke follows


from :
A vB = 7 [T(A v B)]

The conclusion to be drawn from this section is that every logic


f o r m can be expressed by an equivalent one whose sole operation
s y m b o l is the Sheffer stroke. Our interest in this result is only theoreti-
cal,not practical.
2.9 INTERPENDENCE OF LOGICAL OPERATIONS 43

Exercises 2.9
1. For each logic form below, find one logically equivalent to it, but
which uses only the operations v, 1.
(a) [(A B)
-+ C] A A
-+

(b) (A A B) v (B -1 C)
+

(c) [A v (C A D)]
2. Show that v, -+,
and -
B
+

can each be expressed in terms of A and


3. Find a logic form equivalent to the one below, but which uses
1.

only A and 7 .
(A -+ B) v C
4. Express each of the basic logical operations v, A , and
of 7 and -+.
-
in terms

5. Find a logic form equivalent to the one below but which uses
only 1and-.

6. For each logic form below, find one logically equivalent to it, but
which uses only the Sheffer stroke.
(a) A A B
(b) A B
-+

(c) A V 1 B
(d) 1 B A --+

*7. Show that it is not always possible to express a logic form by


means of a logically equivalent one which uses only the logical opera-
tions-., v.
[Hint: Show that you cannot build any contradictions using only the
operations- and v.]

*This denotes a difficult probkm.


Analysis of Inferences
3.1
Sentential Validity
You have probably met the following types of argument before. The
first example constitutes good reasoning, while the second one does
not.

Example 1. If grass is green, snow is white.


Grass is green.
Therefore, snow is white.

Example 2. If x and y are even numbers, x + y is even.


x + y iseven.
Therefore, x and y are even numbers.
Each of these examples is called an inference or an instance of an in-
ference pattern. An inference pattern is a finite list of logic forms pre-
sented as follows:

The logic forms above the bar are called premises while B is called
the conclnsion. (The symbol :. is read "therefore.")
An inference pattern is sententidly valid if and only if
(conjunctionof the premises) -. conclusion
is a tautology. An inference is called sententially valid if its inference
pattern is sententially valid.
46 ANALYSIS OF INFERENCES

Example 3. Consider the inference:


If grass is green, snow is white.
Grass is green.
Therefore, snow is white.
To check it for sentential validity, first symbolize.
G: Grass is green
S: Snow is white.
The inference pattern is

Since
[(G -. S) A G ] -. S
is a tautology (verify that yourself), the inference is
sententially valid.

The sentential validity of an inference depends solely on the way in


which the premises and conclusion are built from their basic sentences
by means of the logical operations. You should convince yourself
that if you substitute sentences for the variable symbols in a sen-
tentially valid inference pattern, and if by doing so you get premises
which are true statements, then the conclusion will also be a true
statement. Thus an inference which is sententially valid constitutes
good reasoning.

Example 4. You observed above that

is a sententially valid inference pattern. An inference


which is an instance of this pattern is:
3.1 SENTENTIAL VALlDITY 47

If cars sing, dogs whistle.


Cars sing.
Therefore, dogs whistle.
This inference, then, is a sententially valid inference.

The preceding example vividly demonstrates that it is only the form


of the inference that determines whether or not it is valid. The fact
that the premises and the conclusion are unrealistic has no bearing
on the matter.
Study the following important example.

Example 5. All countries have armies.


France is a country.
Therefore, France has an army.
Let A be: All countries have armies.
Let B be: France is a country.
Let C be. France has an army.
The inference pattern corresponding to the above in-
ference is:

It is easy to see that the logic form

is not a tautology. Thus the inference is nor sententially


valid. On the other hand, any reasonable person
would probably agree that the inference's conclusion
really does follow from its premises. The reason that a
sentential validity test doesn't help to recognize that
this inference constitutes correct reasoning is that its
correctness is somehow related to the way in which
the premises and conclusion are built from their re-
48 ANALYSIS OF INFERENCES

spective subjects and predicates. B and C have the


same subject while A and C have the same predicate.
Our symbolic resources are not adequate for the job of
revealing this. Logic forms display only the way in
which wmpound sentences are built from their basic
elementary sentences; they do not display the subject-
predicate structure of the elementary sentences them-
selves. The problem of how to recognize the wrrect-
ness of such inferences involves an area of logic
known as "quantification theory," which is beyond
the scope of this book and will not be pursued.

Remember: Inferences which are sententially valid constitute good


reasoning. Inferences which are not sententially valid might represent
good reasoning nevertheless.

Exercises 3.1

A-
1. Test the following inference patterns for sentential validity.
(a) A v B (b) B (c) A -B
BvC
.: -
C-A 1B--I C
:. B v
- -
C B iC
(d) AV B (e) 1A (f) A (B C)
i B v C .. 1 (A A B) ~B-.-IA
A

(g) 1D - 1 B (h) 1 (B v C) -, 1 B
1C D - 1 C
D-A B-1A
BvC .=A-,l D
:. A
2. Determine which of the following inferences are sententially valid.
(a) If the legislature meets today. the executive committee met
yesterday.
The executive committee met yesterday.
Therefore, the legislature meets today.
3.2 BASIC INFERENCES 49

(b) If wages are raised, inflation continues.


If there is a depression, wages cannot be raised.
Therefore, if there is a depression, inflation cannot continue.
(c) If this child is challenged, then he will enjoy learning. If he is
not challenged, this child will be bored with school.
This child does not enjoy learning.
Therefore, he is bored with school.
(d) If other countries develop new weapons, we feel that our na-
tional security is threatened. If we d o not feel that our national
security is threatened, then we will spend more money on social
reform programs.
Therefore, if other countries develop new weapons, then we will
not spend more money on social reform programs.
(e) If the Department of Agriculture is correct, corn blight does
not occur if the crop is sprayed weekly.
Corn blight does occur.
Therefore, if the corn is sprayed weekly, the Department of Agri-
culture is wrong.
(f) Unless protective tariffs are imposed, our balance of payments
will be unfavorable. If our balance of payments is unfavorable, the
foreign aid appropriations will be cut.
Therefore, if protective tariffs are imposed, foreign aid appropria-
tions will not be cut.
3. Give an example of an inference which is not sententially valid,
but which has a true conclusion.
4. Give an example of an inference which is sententially valid, but
which has a false conclusion.
b
5. Give an example of an inference which is sententially valid and
which has false premises and a false conclusion.

3.2
Basic Inferences
Here is a list of three basic inference patterns and their names. Each of
these can be shown to be sententially valid by the method introduced
50 ANALYSIS OF INFERENCES

in the preceding section. These three inferences will be so essential in


later work that you are urged to memorize them.

detachmeat

hypotheticalsyllogism

case inference

Exercises 3.2
1. Verify that the three inference patterns just given are sententially
valid.

3.3
Checking Sentential Validity of
Inferences by Repeated Use of Previously
Proven Inferences
The method introduced earlier for checking the sentential validity of
an inference has the advantage of being conclusive in that it ulti-
mately leads to an answer. Unfortunately, it sometimes leads to in-
volved symbolic manipulations. The technique described now seems
closer to the natural reasoning process and is often shorter. However,
this new method is recommended only if you already suspect that the
inference in question is sententially valid.
In the new technique, inferences that have already been shown to be
sententially valid (in particular those introduced in the preceding
section) are used to prove that the given inference is sententially
valid. The method is based on the following rule:
An inference is sententially valid if the conclusion ap-
pears as the last line in a vertical list of logic forms,
3.3 CHECKING SENTENTIAL VALIDITY 51

each of which satisfies one of the following four condi-


tions:
1. it is a premise,
2. it is a tautology,
3. it is logically equivalent to a logic form preceding it
in the list,
4. it follows from logic forms preceding it in the list
by virtue of a previously proven inference.

Example 1. Verify: A -. B
7B

DEMONSTRATION:
(1) A -. B premise
(2) 1 B > A equivalent to (I) by the law of the
-+

contrapositive
(3) 1 B premise
(4) 1 A Detachment; 3,2.

Ewapk2. Verify: A -, B
C--D
AvC
:. BV D

DEMONSTRATION:

(1) A-B premise


(2)
(3)
(4)
A
C-D
-
B -,(B v D)
(B v D)
tautology
Hypothetical Syllogism; 1.2
premise
(5)
(6)
(7)
D
C -
+(B v D)
(B v D)
(A v C) (B v D)
-+
tautology
Hypothetical Syllogism; 4 3
Case Inference; 3.6
(8) AvC premise
(9) BvD Detachment; 8,7
52 ANALYSIS OF INFERENCES

A chain of inferences like the one in the preceding example is called


a deduction of the conclusion from the premises. Such deductions
resemble mathematical proofs (for example, the proofs of Greek
geometry). This is not to say, however, that all proofs in geometry
are such simple deductions. Analysis of the reasoning used in ge-
ometry has shown that many of the inferences used there depend not
only on the way in which the premises and conclusions are built from
their elementary sentences by means of the logical operations, but
also on the internal (subject-predicate) structure of the elementary
sentences as well. The ancient Greeks did not carry the analysis of
these inferences far enough; the logical analysis of the Greeks was
inadequate for making fully explicit the reasoning used in their own
mathematics.

Exercises 3.3
1. Verify that the following inferences are valid by giving a deduction
from the premises to the conclusion. (The list of basic equivalences
in section 2.2 should be very helpful.)
(b) A -+ B
1C- 1B
C-D

(d) (A
A
- B) v (A -+ C)

:.
B V C

(e) A -
Av B
B (f) B4A
D-C
:. B

(g) 1 A +(B -. 1 C)
7 A V 7 D
lE--B
Switching Circuits
4.1
Representing Switching Circuits
By Logic Forms
In this chapter the relationship between the design of switching cir-
cuits and the algebra of logic forms will be investigated. First you will
see how to represent a switching circuit by means of a logic form. This
is done by making use of some basic properties of electric circuits and
switches.
To each switching circuit you will have to associate a circuit diagram.
Examples of such diagrams are:

The symbol "6 "


represents a terminal of the circuit, while.the
symbol " -" represents a switch in the circuit.
53
54 SWITCHING CIRCUITS

You know that for current to flow from one terminal to the other,
there must be an unbroken path connecting the terminals. A path is
unbroken if and only if all the switches occurring along it are closed.
Using this fact, you can assign to each circuit a logic form which re-
flects the structure of the circuit. The following examples demonstrate
this.

Example 1. Consider the circuit

Switches joined together in this way are said to be con-


nected in series. Assign the logic form A A B to this
circuit because current flows if and only if both
switches A and B are closed.

Example 2. The following two switches are said to be connected in


parallel.

Assign the logic form A v B to this circuit because cur-


rent flows if and only if A is closed or B is closed.
4.1 R E P R E S E N T I N G S W I T C H I N G C I R C U I T S 55

Example 3. The circuit

is an example of a series-parallel combination. Its logic


form is A A (B v C) because current flows if and only if
A is closed,
and
B or C is closed.
In some circuits, one handle operates several switches simultaneously.
Whenever one handle closes or opens several switches together, these
switches are labeled with the same letter.

Example 4. In the circuit

both switches labeled A act in concert and are operated


by the same handle. Its associated logic form is
A A (A v B).
This circuit has three switches, but only two control
handles, an A-handle and a B-handle.
54 SWITCHING CIRCUITS

If one handle simultaneously opens one switch while closing an-


other, it is indicated by labeling the one switch by a letter and the
other by the negation of that letter.

Example 5. In the circuit

the two switches labeled A act in concert but in op-


position to the switch labeled 1 A. All three are con-
trolled by one common handle. This circuit therefore,
has four switches and two control handles, an A-
handle and a B-handle. Its logic form is
A A B A ( ~ A v A ) .

The following table summarizes the procedure used to determine


the logic form which represents a switching circuit.

Asso-
Type of ciated
Diagram Connec- Logic
Representation tion Form Reason
=
series AAB Both A and
B must be
closed for
current to
flow
parallel A or B must
be closed
for current
I toflow
4.1 REPRESENTING SWITCHING CIRCUITS 57

Exercises 4.1
1. Write the logic form associated with each of the following switch-
ing circuits.
58 SWITCHING CIRCUITS

2. Draw the circuit associated with each of the following logic forms:
(a) A A [ ( A A B) v B v ( 7 A A 1 B)]
(b) (A A B) v [(C v A) A 1 B]
(c) A v ( B A l A ) v ( C ~ lBAA)^^ C
(d) E A [ F V G V A ] A [ F V ( ~ E)] G A ~

4.2
Simplifying Switching Circuits
Consider the following switching circuits:

For a given setting of the handles, current flows in the first circuit if
and only if it flows in the second circuit. The first one, however, has
fewer switches. Thus, the first circuit may be called a simplification of
4.2 SIMPLIFY lNG SWITCHING CIRCUITS 59

the second. Actually there is a very practical reason for wanting to


simplify a switching circuit; the smaller the number of switches in a
circuit, the lower the cost of manufacturing it. Imagine how much
money the telephone company could save if it eliminated just one
switch from each bank of switches in the phone system!
In Chapter 2 you learned how to simplify a logic form by algebraic
manipulations. Now you can use the skills you developed there to
simplify switching circuits by doing the following:
1. Determine the logic form associated with the circuit.
2. Use algebraic manipulations to simplify the logic form.
3. Determine the switching circuit associated with the simplified logic
form.

Example. Simplify the following switching circuit and sketch the


simplified circuit.

ANSWER: The logic form associated with this circuit is:


(A v B) A ( 1 A v C) A (B v C).
Now simplify this logic form:
(A v B ) A ( 1 A v C ) A ( Bv C )
commutative law
(A v B) A (B v c ) (~1 A v C )
commutative law
(BvA)A(BvC)A(~AVC)
identity law
(B~A)A(BVC)A(~AVC)A(TAVA)
distributive law
[B v (A A C)] A [ 1A v (A A C)]
distributive law
(B A 1 A) v (A A C)
The switching circuit associated with the logic form
68 SWITCHING CIRCUITS

(B A 1 A) v (A A C)is:

This circuit is a simplification of the circuit with which


you started.

Exercises 4.2
1. Simplify the following circuits.
4.2 SIMPLIFYING SWITCHING CIRCUITS 61
62 SWITCHING CIRCUITS

4.3
Limitations of the Logic Form Method
Of Simplifying Switching Circuits
In the last section logic forms were used to simplify certain circuits.
There are two main drawbacks to this method. First, there is not yet a
"nice" practical way to determine if a switching circuit is simplified
as much as possible. In fact, this problem is still being investigated by
research mathematicians. Also, this method does not take into ac-
w u n t circuits which are not "series-parallel" circuits.

Example. The following is a diagram of a "bridge" circuit.


4.4 DESIGNING A SWITCHING CIRCUIT 63

The circuits which correspond to logic forms are


"series-parallel" circuits, not "bridge" circuits. Thus,
our method would never lead us to a circuit like this.
Consequently, if this were the simplest circuit for a
given situation, our method would fail to reveal that
fact.

Exercises 4.3
1. Find a "series-parallel" circuit which behaves like the following
"bridge" circuit.

4.4
Designing a Switching Circuit Starting
With a Table Which Describes Its Behavior
By examining the diagram of any switching circuit, you can .im-
mediately determine whether current flows for any given setting of the
64 SWITCHING CIRCUITS

switches..Then you can compile a table that shows what the circuit
will do for each setting of the switches.

Example. Display a table describing the behavior of the following


switching circuit for each possible setting of its
switches.

ANSWER: The table should appear as follows:

on I off off 08

t%p+qq
off 1 on

off
on

off
on

off
Now suppose you are given a table like the one constructed in the
preceding example. It may seem surprising, but by using techniques
developed earlier, you can actually design a switching circuit whose
behavior is prescribed by the given table! First think of the table as a
truth table (with "on" playing the role of the truth value t). Then con-
struct the logic form which has this table for its truth table (You
learned how to do this in Section 1.8.) After that, construct the cir-
cuit which corresponds to this logic form. This will be the desired cir-
cuit.
4.4 DESIGNING A SWITCHING CIRCUIT 65

Example 1. Design a switching circuit whose behavior is described


by the following table, and sketch this circuit.

I on I off I off

ANSWER: 1. Restrict your attention to the rows with checks: row 1


and row 4.

2. The sequence of terms corresponding to row 1 is


A, B,
and the resulting conjunction is
A A B.

The sequence of terms corresponding to row 4 is


7 A, 1 B.
and the resulting conjunction is
7 A A l A .

3. The disjunction you are seeking is


(AA B ) v ( l A A iB)
and the diagram of the corresponding switching cir-
cuit is
66 SWITCHING CIRCUITS

The method just outlined will always lead to a switching circuit with
the desired behavior. However, the circuit obtained by this method
may not be the simplest circuit with this behavior.

Example 2. Design and simplify a circuit whose behavior is de-


scribed by the table:

I off I on I on I
1 off 1 off 1 off

ANSWER: The method just learned shows that the table describes
the behavior of the switching circuit corresponding
to the logic form:

Now this can be simplified.

( A * B) V ( 1 A A B)
distributive law
(Av 1A ) A B
identity law

Hence the table also describes the behavior of the very


simple circuit:
4.4 DESIGNING A SWITCHING CIRCUIT 67

Exercises 4.4
1. Design and simplify a circuit whose behavior is described by the
following table:

off I off 1 off 1 off

2. Construct a circuit to control a light switch from three different


locations. [Hint: Make a table to describe the behavior of the circuit.
From the table, construct the logic form and then the circuit.]
3. The five members of the Pookaville city countil are meeting to vote
on a new tax bill. Each man votes in secret by flipping a switch. Design
a circuit so that a light will come on if and only if a majority of the
council votes "yes" for the bill. [Hint: Make a table.]
4. A certain king's five advisors must decide whether to ratify a pro-
posed treaty with the United States. It takes a majority vote of the ad-
visors to ratify the treaty, except that the advisor who is prime
minister has a veto (i.e., the treaty is ratified only if he votes for it).
Design a circuit so that each advisor can vote for ratification by
throwing a switch, and a bell rings if and only if the treaty is ratified.
[Hint: Make a table.]
Set Theory
5.1
Elementary Set Theory
The idea of a set is the foundation upon which most of modern mathe-
matics is built. It is so fundamental that we will not even try to de-
fine it. Synonyms like "collection" or "class" or "family" are often
used to convey the same idea.
The objects which make up a set are called the elements or members
of the set. If an object x is an element of the set A, we write

If x is not an element of A, we write

If every member of A is also a member of B, we say A is a subset of B,


and write

It is easy to see that every set is a subset of itself. Two sets, A and B,
are equal (written A = B) if and only if they have the same members;
that is, every member of A is a member of B and vice versa. In case
A C - B, but A z B, we say A is a proper subset of B and write

This happens when every member of A is a member of B, but some


member of B is not a member of A.

Example 1. The box on the next page should help to clarify the
notions introduced so far.
70 SET THEORY

The set of all


people women
The set of The set of math
teachers at teachers at
Dogwood High Dogwood High
The set of dl 3 The set of all
whole numbers even whole
numbers
The set wnsist-
family ing of Snoopy
and Lucy

airplanes St. Louis planes

In many applications of the theory of sets all the sets which come
into play are built from elements taken from a set U which is specified
in advance and called the universe (for that application). For example,
in plane geometry the universe U is usually taken to be the set of all
points in the plane, and the geometric figures like lines, circles, and
triangles are thought of as sets of such points.
In denoting sets, set braces ( , ) are often used. A set is sometimes
specified by listing its elements inside set braces. For example,
[1,2,8) is the set whose elements are 1, 2, and 8. In specifying a set
this way, it is immaterial in what order the elements are written and
how often they are repeated. Thus

A set can also be specified by stating a property its members will


have. For example,

"the set of all objects which have the property of being


an odd number,"
in short,
"the set of all odd numbers."
5.1 E L E M E N T A R Y S E T T H E O R Y 71

Using x to denote a typical member of this set, we write


( x I x is an odd number)
and read this
"the set of all x such that x is an odd number."
The symbol " ( " is read "such that."
It is convenient to introduce the so-called empty set which has no
members. The symbol $J is used to denote it. It can easily be shown
that $J is a subset of any set A by the following reasoning. To assert
$J C_ A is the same as asserting that for any x, the implication

holds; but this implication holds automatically since for any x, the
left side of the implication is false.
If A and B are sets, the set of elements belonging to either A or B
is called the union of A and B. We write this
A U B.

For example, { 1,2) U (2,3) = ( 1,2,3).


t The set of elements in both A and B is called the intersection of A
f and B. We write this
A n B.
For example, {1,2) fl {2,3) = (21.
The set of elements which is in A but not in B is called the difference
of A and B. We write this

Forexample,[1,2) - (2,3) = (1).


When a universe U is specified and A is a subset of U, U-A is
called the complement of A (with respect to the universe U). The com-
plement of A is denoted by
A'.
For example, if U is the set of all whole numbers, and A is the set of
all even numbers, then A' is the set of all odd numbers.
72 SET THEORY

Example 2. Suppose universe U is the set of all positive whole num-


bers.

Then A U B = ( 1,2,3,4,5,9)
A n B = [1,3,4)
~ n c = $
A - B = (2,5)
A' = ( 6 , 7 , 8 , 9 . . .)

It is possible for the elements of a set to be sets themselves. For


example, for each set A, the power set of A can be formed which has
the subsets of A as its elements. It can be shown that if A has n
elements, the power set of A has 2" elements.

Example 3. If A = ( 1,2,3), the elements of the power set of A are:

Relationships among sets can be visualized by making a type of pic-


ture called a Venn diagram. A few diagrams are given below. Think
of the areas enclosed by the rectangles as the universe U. The rest
ought to be self-explanatory.

A and its complement A '


I Intersection of A and B (shaded)
5.1 ELEMENTARY SET THEORY 73

Union of A and B (shaded) A and B having no members


in common

B is a subset of A A - B (shaded)

Exercises 5.1
Iequal? Why?
2. LetU = {a,b,c,d,e,x,y,z)
Let A = {a,b, c)
Let B = {a,e, x)
Let C = {x, y, z)

It is also given that the objects denoted by the small letters are all
different. Exhibit the following sets.

(a) A' (b) A U B


(c) A - c (d) A n c
(e) (A n cy (f) A U B U C
(g) B - C (h) C - B
74 SET THEORY

3. List the elements in the power set of A = {a, b, c, dl.


4. Given: A = 1, b, C )
B = (e, f,b, g, hl
C = (1,2,c,b)
D = (a, c,e, i )

Basic Identities and Relations of


Set Theory
In this section we have listed the set identities and relations most fre-
quently needed when using set theory to solve mathematical problems.
It is assumed that all the sets appearing in the following list are subsets
of a given universe U.
Basic Identities
law of double
1. (A')' = A
- complement

commutative laws

associative laws
5.2 BASIC IDENTITIES A N D RELATIONS OF SET THEORY 75

6. A n (B u
C) = (A n B) u (A n c)
- 7. A U (B 17 C) = (A u B) f7 (A u C ) distributive laws

de Morgan's laws

idempotent laws

12. A ~ ( A U B =) A
absorption laws
13. A U ( A n B ) = A

identity laws

domination laws

19. A C_ B ifand only if B' C A'


20. A = Bif and only if A Band B C A.
You have, no doubt, been struck by the formal similarity between
these identities and the basic logical equivalences used to manipulate
logic forms. More will be said about this later.
In the example below, one of these identities will be verified for you.
The rest of them can be done in a similar way.

Example. Verify de Morgan's law (for sets).

PROOF: Let x denote an arbitrary object; then,


x E (A U B)' 1 [x E (A U B)] definition of com-
plement
1 [(x E A) v (x E B)] definition of
union
1 (x E A) A 1 (x E B) de Morgan's
law (for logic)
(X E A') A (x E B') definition of
complement
76 SET THEORY

= x E (A' n B') definition of


intersection

Therefore (A U B)' = A' n B'


Exercises 5.2
All the sets appearing in these exercises are subsets of a given uni-
verse U.
1. Under what circumstances are the following statements true?
(a) A n B' = A
(b) A C_ 9
(c) A U B = A ~ B
(d) A - B = B - A
(e) A ~ B = A
( f ) (A u B) n B' = A
(g) (A n B') u B = A u B
2. Verify the following identities.
(a) (A')' = A
(b) A n (B u C) = (A B)n u
(A n c)
(c) A n (A u
B) = A
(d) (A n B)' = A' u B'
(e) (A U B) - (A n B) = (A - B) U (B - A).
(f) A = (A - B) u
(A B)n

5.3
Checking Consistency of Data
Social scientists working for the government spend much time col-
lecting and evaluating data to reach conclusions that help shape
government programs. Private enterprise spends much money on
market research in which data is collected and evaluated to determine
the public's response to a company's products. It may surprise you to
learn that some very elementary set theory can be useful in checking
that the collected data make sense, but this is indeed the case!
5.3 CHECKING CONSISTENCY OF DATA 77

If A is a set, let #(A) denote the number of elements in A. The


following formula, although easy to verify, is important.

Using it, you can now derive a similar formula for #(A U B U C).
The basic idea is to think of A U B U C as the union of two sets,
namely A and (B U C). Now apply the above formula to these two
sets to get
#(A u [B u C I ) = #(A) + #(B u c ) - #(A n [B u CI).
The formula can further be applied to the second term. Moreover,
after applying the distributive law to the third term, the formula can
be applied to its parts. Collecting terms, the result is

Study the two boxed equations! Compare them! If you look at both of
them the right way, you should be able to make a good guess as to how
the formula for four sets ought to look.
In the following two examples you will see how to use the above
formulae in a practical way.

Example 1. In a random survey of 100 people who owned houses or


cars, it was found that 70 people owned a house, and
30 people owned both a house and a car. How many
people surveyed owned a car?

ANSWER: Let H = set of house owners


C = set of car owners
The basic formula is

Substituting the data, the result is


100 = 70 + # ( C ) - 30.
78 SET THEORY

Thus solving for #(C), the answer is


#(C) = 60.
Hence, 60 of the people surveyed own cars.

Example 2. Data concerning the sex, education, and marital status


of the 500 employees of the Sunshine Brewery were
reported as follows:
250 females
156 college graduates
235 married
2 1 female college graduates
73 married females
43 married college graduates
12 married female college graduates
Now the man in charge of assembling the data was a
bit of a boozer, and it was rumored that he made up
the data after having too much to drink. His boss be-
came suspicious and was able to check the data for
consistency as follows:
F = set of female employees
G = set of college graduate employees
M = set of married employees
The data can now be written
250 = #(F)
156 = #(G)
235 = #(M)
21 = #(F 0 G)
73 = #(M n F)
43 = #(M n G)
12 = #(M n F n G)
Putting this data into the previously given formula for
the union of three sets, we get
#(M U F U G) = 250 + 156 + 235 - 21
- 73 - 43 + 12
= 516
5.3 CHECKING CONSISTENCY OF DATA 79

Recall that the company had only 500 employees and


M U F U G being a subset of the set of employees
can have no more than 500 members. This contradicts
the figure of 516 just calculated, so the data is in-
consistent. The data collector, by the way, was trans-
ferred to the tasting department where he seems to be
getting along well.

Exercises 5.3
*l. For the finite sets A, B, C, D, derive the formula:

2. In a survey of 100 welfare recipients, the following facts were dis-


covered:

45 had severe physical handicaps


14 were illiterate
86 lived in substandard housing
5 were illiterate and had severe physical handicaps
39 lived in substandard housing and had severe physical handicaps
12 were illiterate and lived in substandard housing
4 were illiterate, had severe physical handicaps, and lived in sub-
standard housing.

How many of these recipients had severe physical handicaps and lived
in substandard housing, but were not illiterate?
How many were illiterate but were not physically handicapped and
did not live in substandard housing?
How many of those surveyed had none of the above mentioned prob-
lems?

T h i s denotes a difficult problem.


80 SET THEORY

3. In a certain city, drivers lose their licenses if and only if they are
guilty of speeding, failing to stop for a red light, or passing another
car in a "no passing" zone. In 1952,
60% of those with revoked licenses were guilty of speeding,
75% failed to stop for a red light,
70% passed another car in a "no passing" zone,
45% were speeding and did not stop for a red light,
50% were speeding and passed another car in a "no passing" zone,
40% did not stop for a red light and passed another car in a "no
passing" zone.
What percentage of all the drivers who lost their licenses were guilty of
all three violations?
4. The United States Senate is considering three proposals to curb in-
flation: raising income taxes, imposing wage and price controls, and
imposing higher tariffs on imports. Of the 100 senators:
62 support higher tariffs on imports
58 support higher income taxes
24 support wage and price controls
No senator supporting higher tariffs on imports favors wage and price
controls. Of the senators supporting higher taxes, as many favor wage
and price controls as favor higher tariffs. Moreover, each senator
favors at least one of the three proposals.
(a) How many senators favor both higher taxes and wage and price
controls?
(b) How many senators favor only higher taxes? Only higher tariffs?
Only wage and price controls?
Boolean Algebra
6.1
The Abstract Approach
Earlier you were asked to learn a long list of basic logical equivalences,
which were established by means of truth tables. Some of the equiva-
lences in the list could have been derived from others listed there. This
fact was not stressed at the time and now needs to be presented.

Example. The derivation below shows that the absorption law can
be obtained if the commutative, identity, domination,
and distributive laws hold.

AA(AvB)= (AvB)AA commutative law

identity law
(A v B) A (A v F)
distributive law
= A ~ ( B F)
A
domination law
=AvF
identity law
=A
It is not hard to see that a derivation of the "same form" as the one
just given could establish the absorption law for set theory as well.
The following changes of symbolism in that derivation are all that are
needed.
insteadof v
(7 instead of A
= instead of =
9 instead of F
82 BOOLEAN ALGEBRA

Note that verifying the absorption law by the method of the preceding
example yields more information than verifying it by truth tables. The
method really shows that the absorption law holds not only in logic,
but also in set theory, and, in fact, that it holds in any other "algebraic
system" for which the commutative, identity, domination, and dis-
tributive laws hold. If a scientist of a future age were led by the needs
of his science to devise such an "algebraic system," he would be sure
(from our proof) that the absorption law holds in his system too.
This discussion illustrates what is often referred to as the abstract
approach. This is the simultaneous study of a number of systems by
first selecting o r "abstracting" laws which are common to all of them
and then deriving consequences of these laws.
The next section will continue in this spirit. Five laws which are
common to logic and set theory (and other systems) will be selected.
On the basis of these five laws, others will be derived. Algebraic sys-
tems satisfying the five laws selected are called Boolean algebras in
honor of George Boole (1815-1864), who was one of the first people
to use algebraic manipulations in the study of logic.
It would not be appropriate to use the symbolism of logic or the
symbolism of set theory in the development of Boolean algebra since
these are only special cases of Boolean algebras. Instead new symbols,
whose meanings are left unspecified, are used. It is not important to
know which operations and constants these symbols represent. It is
only the laws which govern their behavior that count. Below are listed
the new symbols, their names, and the way they can be interpreted in
logic and set theory.

1 SYMBOL
U
I NAME
CUP
LOGICAL
INTERPRE-

v U
n cap A n
a' complement of a negation of a complement of a
o zero F -
B -

1 one T T U
6.2 DEFINITION OF BOOLEAN ALGEBRA 83

Note: Be careful not to confuse 0 and 1 with the "numbers" 0 and 1


of ordinary arithmetic.
The word "complement" and its symbol were borrowed from set
theory. This is unfortunate as the meaning is not always the same as
in set theory. Nevertheless, this terminology is firmly rooted among
practicing mathematicians, so you will have to live with it.

6.2
Definition of "Boolean Algebra"
If necessary, you should carefully reread the preceding section. Other-
wise, the definition of the notion of a Boolean algebra will not seem
to be well motivated.

Definition. An operation on a set S is a rule of correspondence


which, to each pair of objects a, b of S (taken in that
order), assigns exactly one object in S.

Example 1. Given:
R: the set of real numbers;
+: the usual addition of real numbers;
then "+" is an operation on R which assigns to each
pair of real numbers, a and b, the number a + b.

h m p l e 2. Given:
N: the set ofpositive whole numbers;
-: the usual subtraction of numbers;
then "-" is not an operation on N because even though
a and b are in N, a - b need not be a member of N.
(It might be zero or a negative number.)
84 BOOLEAN ALGEBRA

Definition. A Boolean algebra is a system which consists of a set B


together with two operations u and n, called cup
and cap respectively, and which satisfies the following
five laws:
B1. Commutative laws:
Foralla, b E (8,

B2. Associative laws:

B3. Distributive laws.


For all a, b, c E (8,

a u (b n c) = (a u b) n (a u c)
a n (b u c) = (a n b) u (a n c).
B4. Identity laws:
There exists in (B objects (which we denote by 0, 1 re-
spectively) with the property that
a u 0 = a foralla € I 3 3
and
a n 1 = a forallaE(B
Objects with the property of 0 and 1 are called identity
elements for the operations u and n respectively.

B5. Complementation laws:


For each a E (8, there exists in (8 an object (which we
denote by a') with the property that
aual= 1
a n a' = 0.
Objects with the property of a' are called complements
of a.
6.2 DEFINITION O F BOOLEAN ALGEBRA 85

In some books these laws are referred to as the axioms of Boolean


algebra, while in others they are called the postulates of Boolean
algebra.

Example 3. Given:
a: the family of all subsets of a given universe U;
U: the operation of set union, U;
n: the operation of set intersection, (3;
then (B is a Boolean algebra with respect to the opera-
tions of u and n.9 plays the role of the u-identity 0,
while U plays the role of the n-identity 1.

Example 4. This example shows in precisely what sense the algebra


of logic forms is a Boolean algebra. It is a very so-
phisticated example because the individual members
of this Boolean algebra are themselves sets of logic
forms.

Let I[A] denote the set of logic forms logically equiva-


lent to the logic form A. Let (B denote the family of all
sets which are of the form [AD f a some A. The opera-
tions u and n on 63 are defined for [A], [B] € 63 as
follows:
let

and let

It is not hard to show that (B i s a Boolean algebra with


respect to the operations u and n just defined.
[F] plays the role of the U-identity 0.
(T]plays the role of the n-identity 1.
11 A] is the complement of [A].
Remember The members of (B are not logic forms, but
86 BOOLEAN ALGEBRA

certain sets of logic forms. Two logic forms are in the


same member of B if and only if they are logically
equivalent.

Exercises 6.2
1. In this exercise you have the opportunity to verify that there is a
Boolean algebra with exactly two members.
Let = {p, q), where p and q are two distinct objects. Operations
u and n on CB are defined as follows:

Show that = {p, qJis a Boolean algebra with respect to the opera-
tions of u and njust defined.
Which member of @ plays the role of O?
Which member of B plays the role of l ?
Which member of is the complement of p?
Which member of r$ is the complement of q?

6.3
Duality
Suppose you have a statement in the symbolism of Boolean algebra
and interchange the symbols in that statement as follows:
(a) interchange the symbols u and n
(b) interchange the symbols 0 and 1
The resulting statement is called the dual of the original statement.

Example. Thedual of a u b = c is a nb = c.
Thedualof a u O = a is a n . 1 = a.
Thedualof On 1 = 0 is 1 U O = 1.
If you carefully reexamine axioms B1-B5 in the preceding section,
6.4 SOME T H E O R E M S O F BOOLEAN ALGEBRA 87

you will discover the following curious and important fact:


The dual of each axiom is also an axiom.
This fact has the important consequence that if you prove a statement
using the axioms of Boolean algebra, then you could also prove its
dual. The reason for this is that you could replace each step in the
proof by its dual. The resulting sequence of steps would be a proof of
the dual of the original statement. This remarkable result is referred
to as the principle of duality.

Example. Suppose you were to prove the idempotent law (this will
be done later), a u a = a, from the axioms of Boolean
algebra. Then the principal of duality guarantees that
dual, a n a = a, would also be provable.

6.4
Some Theorems of Boolean Algebra
In this section some consequences of the axioms of Boolean algebra
will be derived. Remember: Each result can be interpreted as a result
of logic, or as a result of set theory.

Theorem I. In any Boolean algebra a,


a) there is at most one element with the property of 0.
That is, there is at most one identity element with re-
spect to the operation U.
b) there is at most one element with the property of 1.
That is, there is at most one identity element with
respect to n.

PROOF: a) Suppose 0 and O* were identity elements with re-


spect to u; we shall show that 0 = O*, i.e., 0 and O*
are the same element after all.
o=ouo*=o*uo=o*
:. 0 = o*
b) The proof in this case is similar to the proof just
given.
I 88 BOOLEAN ALGEBRA

Theorem 11. (IDEMPOTENT LAWS) In any Boolean algebra a, for


alla € a,

PROOF: By using the principle of duality, you need only show


that one of the identities holds. Then the other also
holds automatically. Now prove that a u a = a.

a = a u O B4 (identity law)
= a u (a n a') B5 (complementation law)
= (a u a) n (a u a') B3 (distributivelaw)
=(aua)nl B5 (complementation law)
= a u a B4 (identity law)

Tbeorem 111. (DOMINATION LAWS) In any Boolean algebra (8 ,


for all a E ,

PROOF: Because of the principle of duality, you have to verify


only one of these identities to conclude that both hold.
Provethata u 1 = 1.

I=aua'
= a u (a' n 1)
= (a u a') n (a u 1)
= 1 n ( a u 1)
=(au l)n 1
= a u l
6.4 SOME THEOREMS OF BOOLEAN ALGEBRA 89

Theorem IV. (ABSORPTION LAWS) In any Boolean algebra (B, for


all a, b in@,

PROOF: Because of the principle of duality, you only have to


verify one of these identities to conclude that both
hold. Prove the top one.
an(aub)=(aub)na B1

= a u O Theorem 111
= a B4
You may have noticed that this argument appeared before, in disguise.
G o back to section 6.1 and take a look!

Theorem V. If in a Boolean algebra (B,there are elements a, b, and c


such that both the relations
a u c = b u c

hold, then a = b.

PROOF: a = a u (anc) Theorem IV


hypothesis
B3
hypothesis
B1
B3

hypothesis
Theorem IV
90 BOOLEAN ALGEBRA

'Zbeorem VI. If in a Boolean algebra a,there are elements a, b, and c


such that both the relations
a u c = b u c
aucl=bUc'
hold, then a = b.

PROOF: a = a u O B4
= a u (c n c') B5
= (a u C) n (a u c') B3
= (b u c) n ( a u c') hypothesis
= (b U. c) n (b u c') hypothesis
= bu(cncl) B3
= b u o B5
= b B4

Theorem VII. (UNIQUENESS OF COMPLEMENT) Each element


a of a Boolean algebra (B has at most one comple-
ment.

PROOF: Suppose a' and a* were complements for a. Then, by the


definition of complement,
a u a f =1 aua* = 1
and
anal=O ana*=O
thus:
aual=aua*
and
anaf=ana*
therefore, by Theorem V, a' = a*
6.4 SOME THEOREMS OF BOOLEAN ALGEBRA 91

Tbeorem VIII. In any Boolean algebra B, for any a f B ,


(a')' = a

PROOF: a u a' = 1 and a n a' = 0 by B5. Therefore, a' u a = 1


and a' n a = 0 by Bl. Now this last pair of identities
means that a is a complement for a'. But (a')' is also a
complement for a'. In fact, that is just what the sym-
bolism (a')' denotes. In Theorem VII it was proved
that each element in 63 can have only one comple-
ment. Therefore, a and (a')' must be the same object
that is, a = (a')'.

Exercises 6.4
1. Prove de Morgan's laws. That is, show that in any Boolean algebra
63,for all a, b E
(a u b)' = a' n b' and
(a n b)' = a' u b'
[Hints: 1. Because of the principle of duality, only the top identity has
to be verified. 2. Show that a' n b' has the requisite properties to be a
complement for a u b.]
2. Show that in a Boolean algebra (B, it can happen that a n b
= a n c even though b # c.

3. Show that the associative laws can be derived from the remaining
four laws for a Boolean algebra.
[Hint: Let x = a u (b u c)
y = (a u b) u c.
Then show
a n x = a n y
a' n x = a' n y.
Then use the dual of Theorem VI to conclude that x = y.]
This result was first discovered by E. V. Huntington (1904). It shows
that the associative laws wuld have been omitted from the definition
of Boolean algebra.
Sample Examination

Time: One Hour

1. Make a truth table for the following logic form:


(A A B) v (C -, B)
2. Find a logic form having the following for its table:

3. State in symbols:
(a) Identity Laws (for logic)
(b) Domination Laws (for logic)
(c) Distributive Laws (for logic)
(d) Absorption Laws (for logic)
4. Simplify:
[(A A B) v (A A B A C)] v 1A

5. Test for tautology (without using truth values):


[(A A B) -. A] A A
6. Test for contradiction (without using truth values):
(A A 1B)+ B
94 SAMPLE EXAMINATION

7. Simplify:

*-4-72 .

8. Writha logic form equivalent to

but which uses only the operation symbols 7 ,v. /

9. Write a logic form equivalent to

but which has the Sheffer stroke as its only operation symbol.
10. Symbolize the following argument and check it for sentential
validity.
"Our mayor would keep his promises only if he were a man of
integrity. But our mayor doesn't do this. So he lacks integrity."
11. In Cheerville, Illinois, all the 200 male inhabitants are devoted to
wine, women, or song.
A poll showed:
100 are devoted to women
100 are devoted to wine
100 are devoted to song
20 are devoted to women and song
50 are devoted to women and wine
50 are devoted to wine and song
(a) How many are devoted to wine, women and song?
(b) How many are devoted to wine and song but not to women?
Answers
EXERCISES
SECTION 1.2
1. (a) It is not the case that geeks are foobles. ("Geeks are not
foobles" is also an acceptable answer.)
(b) Geeks are foobles and dobbies are tootles.
(c) Geeks are foobles and it is not the case that dobbies are tootles.
(d) If it is not the case that geeks are foobles, then dobbies are
tootles.
(e) It is not the case that if geeks are foobles, then dobbies are
tootles.
(f ) Geeks are foobles if and only if dobbies are tootles.
(g) If it is not the case that geeks are foobles or if dobbies are
tootles, then geeks are foobles.
(h) Geeks are foobles and dobbies are tootles if and only if it is not
the case that dobbies are tootles.
2. (a)
(b)
(c)
A -
A AB

iA-B
B ,

(d) A -. (B V C)
(e) (B A C) -. A
(f)
(g)
-lBo A
1 (A - B)
% ANSWERS

4. (a) t o r f
(b) lmo truth value
(c) t
(4 f
(e) t o r f

SECTION 1.3
1. (a) B A A
(b) 7 DA C
(c) C-7 D
(d) AVB
(e) C--D
2. (a) MA J
(b) (M A J) A T
(c) (M v J) -, T
(d) M -- J
(e) i M v J
(f) J A M
(g) T -- -lJ

SECTION 1.4
1. The sentences written symbolically are:
(a) D -+ R
(b) T -+ 1R
(c) R - D
Their converses (in English) are:
(a) If Ollie roars, then Ollie is a dragon.
(b) If Ollie does not roar, then Ollie is toothless.
(c) Ollie is a dragon only if Ollie roars.
Their converses, written symbolically, are:
(a) R.-- D
(b) 7 R -,T
(c) D - - R
EXERCISES 97

Their contrapositives(in English) are:


(a) If Ollie does not roar, then Ollie is not a dragon.
(b) If Ollie roars, then Ollie is not toothless.
(c) If Ollie is not a dragon, then Ollie does not roar.
Their contrapositives,written symbolically, are:
(a) l R - 1 D
(b) R -+ 1T
(c) - I D - 1 R

2. (a) t
(b) t
(c) t o r f

3. (a) A possible example is: If 2 + 2 = 4, then 2 2 = 4. Of course,


you could concoct many correct answers.
(c) A possible example is: If 2 + 2 = 5, then 2 2 = 4. Many
correct examples can be constructed.

SECTION 1.5

(b) 1A I B I C 1 ( C B A B I
-

98 ANSWERS

t f t t
t f f t
f t t t
f t f t
f f t t
f f f t
EXERCISES 99

2. (a) t
(b) t
(c) t
(d) indeterminable
3. (a) Let S be: The stock's value rises.
Let D be: A dividend is declared.
Let M be: The stockholders will meet.
Let B be: The board of directors summons the stockholders.
Let C be: The chairman of the board resigns.
Then the logic form corresponding to the sentence is:
(S v D)- [M - (B A 1 C)]
(b) f; indeterminable since the truth value of M is unspecified.
Horace Gladstone Klunker

4' H G K ~ G A K 1H-+,K KA(,HV.G) .


t t t f t f
t t f f t f

(a) Horace and Klunker lied.


(b) Horace and Klunker are innocent; Gladstone is guilty.
(c) Horace and Klunker are guilty; Gladstone is innocent.
5. (a) Diplomat B is the spy.
(b) Diplomat A is the spy.
6 Yes. The logic forms correspondingto the complied facts, in order,
are:
A: F [[(G A 1 (L v R)] v [L A
-+ 1 (G v R)]]
v [ R A i(L v G)]
B: (~GA-IL)-.F
100 ANSWERS

C: 1 R + [(F A L) v (F A G ) v (G A L)]
D: (L A G) -, 1 R.
Then the truth table discussed in the hint is:

By examining the checked rows, you should be able to see that the
registrar followed his instructions.

SECTION 1.6

tautology
EXERCISES 101

contingency

contingency

contingency

(f) A I B (I,(AAB)-(,Av-IB) tautology


t ItII t

SECTION 1.7
1. Form the conjunction

A l ( 1 A A 1 B)
and examine its truth table.
102 ANSWERS

Hence the given logic forms are sententially inconsistent.


2. (a) Let A be: A recession will occur;
Let B be: Unemployment will decrease;
Let C be: Wage controls will be imposed.
The sentences given can then be symbolized as follows:
Av(7B-C)
iA--B
C-. 1B

These sentences are sententially consistent.


(b) Let A be: Imports increase.
Let B be: Exports decrease.
Let C be: Tariffs are imposed.
Let D be: Devaluation occurs.
The sentences given can then be symbolized as follows:
(A v B) + (C v D)
CW(A A 1 D)

B--(iCv 1A)
1DV(CAB)

They are sententially consistent.


3. Their advice is inconsistent; they can't all be correct. When told
about this, Fox muttered "A foolish consistency is the hobgoblin of
small minds."
SECTION 1.8
1. (a) (A A B) v (A A i B) v ( iA A B)
(b) (A A 1 B ) V ( l A A 1 B)
EXERCISES 103

(d) ( A A B A C) v ( A A B A 1 C) v ( A A 1 B A C )
v ( ~ A A B A C ) V ( ~ A A B A - I C ) V B( A~ CA) A ~

3. The truth table for the unknown logic form should be:

The table is constructed s o that the value in Blanco's column agrees


with the value in the B column and s o that the logic form column
agrees with Blanco's column only for those rows where A has value t.
Therefore, the unknown logic form is

(A A B) V ( 1 A A 1 B).

The question should be:


"Is it the case that either you tell the truth and the left-hand branch
leads t o the settlement, o r you lie and the left-hand branch does not
lead t o the settlement."

SECTION 2.1
104 ANSWERS

A V B + l ( l B ~ 1 C )
The symbol means that the logic forms are not equivalent.

t f t t
t f f t

f f t t
f f f t
EXERCISES 105

The logic form:


( A A B A C ) V ( A AB r~r C ) v ( A ~1 B A ~ C )

is logically equivalent to A -, (B + C) because it has the same truth


table.

SECTION 2.3
~ . ( ~ ) A v B1 -1 A v B
= lA-- B
106 ANSWERS

(b) 1 ( A - B) = l ( i A v B)
= 11AAlB
E A A l B

(d) (A A B) v ( 1A A C)
= [A v ( 1 A A C ) ] A [B v ( 1 A A C)]
= ( A v 1 A) A ( A v C ) A ( B V1 A) ~ ( B v c )
= (A v C) A (B v 1 A) A (B v C)
= (B v 1A) A (A v C) A (B v C)
= (~AvB)A(AvC)A(BVC)
= (A B) A (A v C) A (B v C)
= (A B) A [(A A B) v C]
-+

= (A B) A [C v (A A B)]
-+

2. ( a ) i A v ( A v B ) =(iAvA)vB
= 1A V A
=T
(b) ~ ( ~ B V A ) = B A ~ A
( c ) T A A ( A V B )= ( ~ A A A ) v ( ~ A A B )
= 1AAB
(d) (A A B) v (A A 7 B) = A A (B v 1 B)
iA

(e) i ( A A 1( A v B ) ) = i A v 1( i ( A v B ) )
= i Av(AvB)
(1AvA)vB
(iAvA)
=T
(f) Ar(AvBvC) (AAA)v(AAB)v(AAC)
= Av(AAB)v(AAC)
= A v [A A (B v C)]
= A
(g) [ ( l B v A) A A ]- + A
-
= [A A ( ~ vBA ) ] - + A
[A A (A v 1 B)]
-/,-.A
-+ A

=T
EXERCISES 107

( h ) (A V B ) A[(B VA) AC] %e (A VB) A[(A v B) AC]


= [(A v B) A (A v B)] A C
= (AvB) A C
(i) l [ A v ~ ( B A C ) =
] ~ A A [I ~ ( B A C ) ]
=~AA(BAC)
( j ) A A ( ~ A v B ) = ( A AA) v~ ( A A B )
~ A A B

(m) [A A (B A C)] v [ i A A (B A C)] = (A v i A) A (B A C)


BAC
( n ) First note that the dotted expression on the top line is a tau-
tology.
(E A D) ~ ( [(Ai + B)-(1B- i A] -, C )

-
= (E A D) A ( 7 [A v iA] C) +

E (E A D) A ((A v 7 A) v C)
(E A D) A (A v 1 A)
= (E A D).
3. (a) it her freedom of the press is not an important safeguard of
liberty or, in protecting it, our courts have not played a major
role.
(b) Although a politician seeks the presidency and he has sufficient
financial backing, he cannot afford to appear on nationwide
television frequently.
(c) A man has no self respect although he is contributing to a
better society.
(d) We can halt pollution although we do not act now.
(e) Although a man cannot join the union, if he must relocate his
family, he can find a job in that factory.
(f) A worker can share in the company profits, but if he demands
fewer fringe benefits, then he does not work harder.
108 ANSWERS

[Remember, there are many correct answers to this question; we


have just presented some possible answers.]
4. No, for example

5. A A ( A v B ) (A V F ) A(A v B )
= A V (F A B)
=AvF
=A

-
6. [T -,(A v E)] A [ 1 E -, i (W A T)] A [(A A i E)
= [T (A v E)] A [(A A 1 E) 1 TI A [(W A T) El
-+
1 TI
+

-+

= [ l T v ( A v E ) l ~ [ l ( A E~ )iv l T ] A [ ~ ( W A T ) V E ]
= [ i T v A v E ] ~ [ i A v E v i ~ ~ [ il TWv vE ]
= [ ~ T ~ E ~ A ] A [ ~ T ~ E ~ T~v EA] ] A [ ~ W V
[ [ l T v E v A ] ~ [ i T v E iv A ] ] ~ [ l W v Tl v E ]
= [ [ ( i T v E) v A] r , [ ( i T v E) v i A ] ] A [ i W v ( 1 T v E)]
= [ ( l T v E) v (A A i A)] A [ ( 1 T v E) v W]
= ( T T V E ) V [ ( A A1 A) A W ]
= (-ITvE)v[FAW]
=(7TvE)vF
= (iTvE)
=T-E
A person pays taxes only if he earned $1,000 in the past 12 months.
7. The given set of rules corresponds to the logic form
(W -. H) A [(H A F) -,W] A (F + 1 W).

Now simplify this logic form:


(W - H) A [(H A F) --+W] A (F 7 W)
-+

= ( 1 W V H ) A [ T ( H AF ) v W ] ~ ( l F v W) i
= ( l W v H ) ~ [ i H vi F v W ] ~ ( i F v W) i
= [ ( i w v H ) v ( F ~ i F ) ] ~ I i Hi Fv v W ]
~ [ ( i F1 v W ) v ( H A i H)]
= ( l W v H v F ) ~ ( i W v H v i F ) ~ ( i H vF vl W )
~ ( i F iv W v H ) ~ ( i F v i W vl H )
idempotent law
= ( i W v H v F ) ~ ( l W v H vi F ) ~ ( i H vi F v W )
~ ( i F vW i v iH)
EXERCISES 109

r ( i W v H v F ) ~ ( l W v H v7 F ) ~ ( l H v l F v W )
~ ( 1 H 1 v F v 7W)
= [ ( l W V H ) A ( F A1 F ) ] A [ ( ~ V
H I F ) A ( W A i W)]
r ( i W v H ) * ( i H v l F)
5 (W H) A (H
-+ 1 F)
+

Thus the simplified rules are:


(a) The warriors shall be chosen from among the hunters.
(b) N o hunter shall be a farmer.

Section 2.4
1. (a) is (f) is not
(b) is (g) is
(c) is not (h) is not
(d) is (i) is not
(e) is

Section 2.5
i A-Br 7 1 AvB
= AVB
i(lBvA)= B A ~ A
(A A B) v (A A 1 B) = (A + B)(A + B')
= AA + BA + AB'
+ BB'
i (A A i ( A v B ) ) =
A v [ i i (A vB)]
i
= 1AvAvB
A - B = ( A - B)A ( B + A)
( ~ A V B ) A ( ~ B V A )
A A [B v (C A i D)] = A + [B(C + D')]
= A + BC + BD'
(A + B) A [ C v (A A B)] = A'B + [C(A + B)]
= A'B + CA + CB
(A A B) v [(A A C) v (A A 1 B)]
= (A + B)(A + C)(A + B')
P (A + B)(AA + CA + AB' + CB')
= AAA + BAA + ACA + BCA + AAB' + BAB'
+ ACB' + BCB'
110 ANSWERS

SECTION 2.6
1. (a) tautology
(b) tautology
(c) tautology
(d) not a tautology
(e) tautology
(f) not a tautology
(g) not a tautology
(h) not a tautology
(i) not a tautology

SECTION 2.7
1. (a) is
(b) is
(c) is not
(d) is
(e) is
(f) is not
(g) is not
(h) is
2. We work in ordinary notation; you may prefer to use arithmetical
notation.
(a)A--B= i A v B
(b) A V B

(d) A -
(c) 1 ( A v B ) = 1 A A 1 B
B = (A -, B) * (B - A)
= ( 1 A v B ) A(IBVA)
= [ ( l A v B ) ~ Bl ] v [ ( ~ A v B ) A A ]
= ( ~ A ~AB ) v ( B A ~B)V(TAAA)V(BAA)
=(-IAATB)V(BAA)
(e) A A B A C
(f) (A v B) A (B -- C)
= (AvB)*(lBvC)
= [(A v B) A 7 B] v [(A v B) A C]
= (A A 1 B) v ( B A 1 B) v ( A A C ) v ( B A C )
= (A A 1 B) v (A A C) v (B A C)
s '<I'
-
EXERCISES 111

(g) [ i ( B - + C ) ] - A = 7 [ l (
lBvC)]vA
= 1B v
CvA
( h ) i [(B-.C)-+A] = i [ i ( l B v C ) v A ]
( ~ B V C ) 1A A
( ~ B iAA ) v ( C A 1 A)
( i ) (A v B) A [(B v A) A C ] = (A v B) A ( A v B) A C
= (AvB)AC
= (A A C) v (B A C)
( j ) A ~ [ ( Ai + B) v C ] = A A [ ( A v B ) v C ]
= (A A A) v (A A B) v (A A C)

(k) (A
=
-B)
+ [(B
-+ C) -+

(lAvB)-+[(lBvC)-+(lAvC)]
-
= AV(AAB)V(AAC)
(A -* C)]

= (iAvB)v[(lBvC)-(lAvC)]
i
= (iAvB)v[l(iBvC)v(lAvC)]
i
= ( A A ~ B ) v ( B A ~ CA )v VC ~

SECTION 2.8
1. (a) not a contradiction, but is a contingency
(b) not a contradiction, but is a contingency
(c) contradiction
(d) not a contradiction, but is a contingency
2. (a) tautology
(b) tautology
(c) tautology
(d) contradiction
3. Let A be: The XYZ company builds a new factory
Let B be: Some of the XYZ company's machinery must be re-
placed
Let C be: The XYZ company can obtain a loan
Let D be: The XYZ company will be threated with bankruptcy
The sentences in the exercises correspond to the following logic
forms:
(A v B) -+ (C v D); C .- (A A 1 B)
B-+(lCV 1 A); DV(BAC).
112 ANSWERS

Now take the conjunction of these and reduce it to a disjunctive


normal form. You will see that it is not a contradiction, so the sen-
tences are sententially consistent.

1. (a) [(A - SECTION 2.9


B) -+ C] A A =
=
1 [ 1[(A
B)
1[ i [ i (
C] v 1 A]
-+ -+

i A v B ) v C ] v i A]
(b) ( A A B ) v ( B - . 1 C) = 1 ( 1 A v 1 B ) v ( l B v 1 C )
(c) [A v (C A D)] -+ B 7 [A v (C A D)] v B
= i [ A v i ( i C vD ~ )]vB
2. AvB=i(iArriB)
A+B= 1AvB
= l ( 7 7 A A T B)
= 1( A A l B )
A - B = (A+B)A(B-+A)
= 1( A A l B ) A l ( B A l A )
3. ( A + B ) v C 1 [ l ( A + B ) A 1 C]
= 1 [ 1 ( l ( A A 1 B)) A l C ]
= l [ A A l B A l C ]
4. AvB=lA-+B
A A B El ( 7 A v l B )
= l(llA+lB)
= l(A+lB)
A - B = ( A + B)A ( B - A)
= 1 [(A B)
5. A A (B v C) = 1 [A
1 (B
+

= -I [ A -
-+

1
-I
+

(B v C)]
(iB-C)]
A)]

6. (a) A A B = 1 [ 1(A A B)] = 1 (A I B) = (A 1 B) (A I B) I


(b)A-B= l A v B = i ( A ~ i B ) r A l ( i ~ )
= A((B1B)
( c ) A v ~ B = ~ B v A P ~ ( B A ~ A ) = B ( ( ~ A )
= B (A / A ) I
( d ) l B - - A = ( B v A ) = i ( T B A ~ A 7 ) B~ l - t ~
= (B I B) (A I A) I
7. Stronger hint: Use an indirect proof. Suppose a contradiction
could be built from and v. Then there would be one of smallest
-+

length.
7

EXERCISES 113

SECTION 3.1
not sententially valid
sententially valid
sententially valid
sententially valid
sententially valid
sententially valid
sententially valid
not sententially valid
2. (a) not sententially valid
(b) not sententially valid
(c) sententially valid
(d) not sententially valid
(e) sententially valid
(f) not sententially valid
3. Hint: Any inference of the form
A-B
B
:. A
in which A is a true statement will do.

4. Hint: Any inference of the form


A-B

in which B is a false statement will do.


5. Hint: Any inference of the form

in which A and B are true statements while C is a false statement will


do.
114 ANSWERS

1.
SECTION 3.2

11. Note that [(A


- -
-- - -
I. Note that [A A (A B)] B is a tautology.
B) A (B -. C)] -+ (A -+ C) is a tautology.
111. Note that [(A C) A (B C)] [(A v B) -+ C] is a tau-
tology.

SECTION 3.3
1. (a) A
A -. (A v 1 B) taut.
A v l B
7 BVA
B-A
(b) 1 C- 1 B
B-+C
A-B
A-C
C-+D
A-D
(c) c
C - i B
1B
A-B
1B- 1A
1A
(d) (A -B) v (A
(lAvB)v(l AvC)
C)
-+

A
A
-
1Av(BvC)
(B v C)

BvC
(e) A v B
1A-B
A-+B
(lAvA)-+B
1( 1 A v A ) v B
(AA 1 A ) V B
B
(f) B - A
1A- 7B
EXERCISES 115

B v D
1 B--D
1A - D
D--C
A-c
1
A v C
(g) 7 A V l D
D - 1A
D
1A
1A-(B-1C)
B--1 C
1 E-B
1 E- 1C
C-E

SECTION 4.1
1. (a)(A v B) A -I A
(b)A A (B v 1 B)
(c)(A v B v 1 C) A (A v C)
(d)A v ( ~ B lAC ) v ( D ~ 7B ) v ( C A A )
(e) A A [(B A i C) v (C A ( D v i A))]
116 ANSWERS

SECTION 4.2
1. (a) A A (1 A v B ) = (A ~ 7 A ) v ( A A B )
I AAB

-
(b) (A

=
A B) v [(C v A) A 1 B)]
( A A B) ~ ( C 1 AB ) ~ ( A 1
( A A B)v(A A 1B) ~ ( C 1
A B)
AB )
[(A A (B v 7 B)] v (C A 1 B)
= Av(CA iB)
EXERCISES 1 17

(c) ( B A C ) V ( A Ai B ~ c ) v ( l AA 1 B ~ c )
= ( B ~ c ) v [ ( A v~ A ) A ( ~ B A C ) ]
= (BAC)V(~BAC)
= ( B v iB ) A C
=c

(d)(lAv ~ B ) A ( ~ A V B ) A ( A V B )
I [ l A v ( B A 1B)] A ( A v B )
= ~AA(AvB)
=(~AAA)V(JAAB)
= 1A A B

(e) A A [(A A B) v B v ( 7 A A 1 B)]


= A A [ B v ( ~ A lAB ) ]
= A A [ B v ~ A ]
= (A A B) V (A A 1A)
= AA B

(f) ( A v B v C V D ) A ( A V B V D ) A ( A V C )
absorption
(AvBvD)A(AvC)
A v [(B v D) A C ]
118 ANSWERS

(g) [C A (A v iB)] v [ -I C A A] v [(C v -I B) A iC]


= [ C A ( A V ~ B ) ] V [ ~ C A A ] V [ C A ~ C ]C]V [ ~ B A
= ( C A A ) V ( C A ~ B ) V ( ~ C A A ) V ( -A I 1CB )
5 [(C v 7 C) A A] v [(C v -I C) A -I B]

= AV 1 B

(h) (A A B) v ( 1 A A C) v (B A C)
= (AAB)V(CAB)V(-IAAC)V(~AAA)
5 [(A v C) A B] v [ 1 A A (C v A)]
= [(A v C) A B] v [(A v C) A 1A ]
(A v C ) A (B v 1A)

SECTION 4.3
-
EXERCISES 119

SECTION 4.4
1.

2. Stronger hint: Finish filling out the table below. T o say that each
switch can control the circuit is the same as saying that if, in two given
rows, exactly two of the letters A, B, C get the same value, then the
right-hand entries of those two given rows should be of opposite value.
This information should enable you to complete the table.

ml I
I 1 I1

on I on I off 11

off on
off off off

3. Stronger hint: Make a table letting A, B, C, D and E denote the


switches controlled by each of the councilmen. If, in a given row, three
o r more of these letters get the value "on," then the right-hand entry
for that row should get the value "on."

4. Here A denotes the button pushed by the prime minister, while


I20 ANSWERS

B, C, D, and E denote the buttons controlled by each of the other four


advisors.

This circuit can, of course, be simplified.

SECTION 5.1
--

EXERCISES 121

4. (a) true
(b) true
(c) false
(d) true
(e) true
(f) true
(g) false
(h) false
(i) true
(j) true
(k) false
(1) false

SECTION 5.2
1. (a) B C_ A' (or, equivalently, A -
C B')
(b) A = 9
(c) A = B
(d) A = B
(el A C_ B
(f) A C_ B' (or B C_ A')
(g) always
122 ANSWERS

2. (a) x E (A')' = 1(x E A')


= i ( i (X E A))
-
= xf A
(b) x E (A n (B u c)) = (X E A) A [X E B v x E CI
= (xEAAxEB)
v(xE A A x E C )
= (XE ~n B ) V ( X E A ~ c)
x E [(A n B) U (A n C)1
=
(c) X E A ~ ( A U B ) = ( X E A ) A ( X E A ~ X E B )
E X E A
(d) x E (A n BY = 1(X E (A n B))
= ~(xEAAxEB)
= i(xEA)vi(xEB)
=xEAfvxEB'
= xEA1UB'
(el x E ((A u B) - (A n B))
= X E ( A U B ) A ~ ( X E A ~ B )
~ ( x E A v x E B ) A ~ ( x E A A x E B )
= ( x EA v x E B ) A [ i ( x E A ) v i ( x E B)]
= [(XEAVXEB)A~(XEA)]~
[(x E A v x E B) A A i(X E B)]
= [ x E B A l ( x E A 1 ~ [ x E A ~A( x E B ) ]
E x E B - A v x E A - B
= x E [(B - A) U (A - B)]
(f) x E (A - B) u (A n
B)
= x E P - B v x E A n B
[xEAA~(xEB)]v[xEAAxEB]
(X E A) A [ 1(X E B) v (X E B)]
=xEA

SECTION 5.3
1. Hint: By reassociating, the union of more than two sets can always
be treated as just the union of two sets; e.g.,

Keep applying the formula for two sets


EXERCISES 123

over and over, using the distributive law where necessary. The answer
finally is:

Closely examining the form of this answer, you should be able to


make a good guess as to how the formula ought to look for calculating
the number of elements in the union offive sets!

3. 30"/,. Hint: Let M be the set of people losing their licenses, and let
m = #(M). Then if
S is: the set of speeders,
R is: the set of people not stopping for a red light,
P is: the set of people passing in "no passing" zones,
#(S) .6m, #(R) = .75m, #(P) = .70m,
=
#(S = .45m, #(S n P) = SOm, #(P
n R) n R) = .40m,
and
#(MI = #(S U P U R).
4. (a) 22
(b) 14,40,2

SECTION 6.2
1. To show that CB is a Boolean algebra, check that (B satisfies axioms
B1-B5 by actually computing all the appropriate expressions.
"p" plays the role of 0
"q" plays the role of 1
P' = q
9' = P
\

124 ANSWERS

SECTION 6.4
1. Because of the principle of duality, verify only

(a u b)' = a' n b'


(i) First note (a u
b) n (a' n b') = [(a u b) n a'] n b'
= [(a n a') u (b n a')] n b'
= lo u (b n a91 n b'
= (b n a') n b'
= (a' n b) n b'
= a' n (b n b')
=a1nO
= 0
(ii) Now note(a u b) u (a' n b') = a u [b u (a' n b')]
= a u l(b u a') n (b u b')]
= a u [(b u a') n 11
= a u (b u a')
= a u (a' u b)
= (a u a') u b
= l u b
= 1

From (i) and (ii) you can see that a' n b' has the requisite properties to
be the complement of a u b. Hence, by Theorem VII, (a u b)' =
a' n b'.

2. Let (B be the Boolean algebra consisting of the subsets of a set X,


with u and n being set union and intersection respectively. Let A and
B be two distinct subsets of X. Then.
SAMPLE EXAMINATION 125

Moreover:
a' n x = a' n [a u (b u c)]
= (a' n a) u [a' n (b u c)l
= 0 u [a' n (b u c)l
= a' n (b u C)
and
a' ny = a' n [(a u b) u cl
= [a' n (a u b)l u (a' n c)
= [(a' n a) u (a' n b)] u (a' n c)
= [O u (a' n b)l u (a' n c)
= (a' n b) u (a' n c)
= a' n (b u c)
Hence a' n x = a' n y. So by the dual of Theorem VI, x = y.

SAMPLE EXAMINATION

Problems of this type are discussed in Section 1.5.


2 . ( A ~ lB A ~ C ) V ( ~ A A B A ~ C )
Problems of this type are discussed in Section 1.8.
126 ANSWERS

3. (a) Identity Laws


B A T =B
BvF B
(b) Domination Laws
B v T = T
B A F ~ F
(c) Distributive Laws
A A (B v C) = (A A B) v (A A C)
A v (B A C) = (A v B) A (A v C )
(d) Absorption Laws
AA(AvB)PA
Av(AAB)=A
Consult Section 2.2.
4. Start with
[(A A B) v (A A B A C)] v 1 A
Apply absorption law to the expression in the square brackets, treat-
ing A A B as a single quantity, to get
[ A A B] v i A
commute
1A v[A AB]
distribute
(iAvA)rr(iAvB)
identity
1 AVB
Consult Section 2.3.
5. Reduce to conjunctive normal form. Note that in the conjunctive
normal form, at least one conjunct fails to contain a letter and also its
negation. Thus, the logic form is not a tautology.
Consult Section 2.6.
6. Reduce to disjunctive normal form. Note that at least one disjunct
fails to have a letter and also its negation. Thus, the logic form is not a
contradiction.
Consult Section 2.8.
SAMPLE EXAMINATION 127

7. The logic form for this circuit is

This can be manipulated into

Thus the simplified circuit is

T BC
- b
Consult Section 4.2.
8. (A A B) + C
arrow law
1(AAB)VC
de Morgan
1A v 1B v C Answer. (This, however, is not the only correct
answer.)
Consult Section 2.9.
9. A + 1 B
1 A V l B
1 (A A B)
A I B Answer. (This, however, is not the only correct answer.)
Consult Section 2.9.
10. Let P be: Our mayor keeps his promises.
Let I be: He is a man of integrity.
The argument in symbols is

Form the logic form


[(P - I) A 1PI + 1I

This is not a tautology. Thus, the argument is not sententially


valid.
Consult Section 3.1.
ID ANSWERS

11. Let W be: Set of all those men who like women.
Let G be: Set of all those men who like wine (grapes).
Let S be: Set of all those men who like song.
Consulting Section 5.3, find the identity

Substituting given values, the result is

Solving for #(W f l G n S), you get


#(W n G n S) = 20 Answer topart(a).
To solve part (b) proceed as follows:
(G n s) = (G n s ) n cw u wl)
identity law
= ( ~ n s n w ) u ( ~ n s n w ~ )
distributive law
Since the two sets which you are unioning on the right are disjoint,
you can simply add up the number of members in each to get

Substituting the value given for #(G n S) and the value calcu-
lated in part (a) for #(G fl S (7 W), you have
Suggested Books for
Further Reading
Elementary Bowran. A. P. A Boolean Algehra. New York, Macmillan.
1965.
Dubisch, Roy. Latrices to Logic. New York. Blaisdell, 1963.
Hill, Shirley, and Suppes, Patrick. Firsr Course in Mathemarical
Logic. New York, Blaisdell. 1964.
Quine, Willard Van Orman. Elementary Logic. New York.
Blaisdell, 1964.
Tarski. Alfred. Inrroduction ro Logic. New York, Oxford Uni-
versity Press, 1965.

Advanced Hohn, F. E. Applied Boolean Algehra. New York, Macmillan,


1960.
Kneebone, G . F. Marhemarical Logic and rhe Foundations of
Mathema~ics.D . Van Nostrand, New York, 1963.
Lightstone, A. H . The Axiomatic Merhod. Prentice Hall, Engle-
wood Cliffs, New York, 1964.
Mendelson, Elliott. Boolean Algehra and Switching Circuits.
McGraw-Hill, New York. 1970.
Stoll, Robert R. Set Theory and Logic. W . H. Freeman, San
Francisco. 1961.
Index
Absorption laws, 23, 75 Data, consistency of, 76-79
Abstract approach, 82 De Morgan's laws, 23.75
Algebraic manipulations, 24-26, 30 Deduction, 52
Algebras, Boolean, 81-91 Detachments, 50
Alternative translations, 7-8 Differences, 71
And, I Disjunction, 1
Arrow law, 23 Disjunctive normal forms, 35-40
Associative laws, 23, 74 Distributive laws, 23, 75, 81
Axioms (Boolean), 85 Domination laws, 23, 75.81
Double arrow law, 23
Double complement, law of, 74
Basic equivalences, 23-24
Double negation, law of, 23
Basic inferences, 49-50
Duality, 86-87
Basic logical operations, 1, 15,40-42
Duals, 86
Bi-implication, I
Boolean algebras, 8 1-9 1
Elements, 69
Braces, set, 70
Empty sets, 71
Equal, 69
Capital letters, 9-10 Equivalence, logical, 2 1-22.24-26, 5 1
Caps, 84 Equivalences, basic, 23-24
Case inferences, 50
Cause-and-effect relationships, 4 Forms:
Circuit diagrams, 53-56 conjunctive normal, 29-35
Circuits, switching, 53-67 disjunctive normal, 35-40
Commutative laws, 23,74,81 logic, 9-1 1, 14, 16-18,29, 51
Complements (set theory), 7 1
Complements (Boolean), 84 Hypothetical syllogisms, 50
Compound sentences, 1, 14-1 5
Conclusions, 45 ldempotent laws, 23,75
Conjunction, l,45 Identity (Boolean), 84
Conjunctive normal forms, 29-35 Identity laws, 23, 75, 81
Conjuncts, 29-30, 33 If and only if, 1
Consistency, sentential, 15 I f . . .then.. . , 1
Consistency of data, 76-79 Implication, 1.8
Contingencies, 14 Inconsistency, sentential, I5
Contradictions, 14, 38 Inference patterns, 45
Contraposition, law of, 23 Inferences, 45
Contrapositives, 8-9 basic. 49-50
Converses, 8-9 case, 50
Cups, 84 previously proven, 50-52
INDEX 132

Instances of inference patterns, 45 Sentences, compound, 1, 14-15


Intersections, 7 1 Sentential consistency, 15
Sentential inconsistency, 15
Law of contraposition. 23 Sentential validity, 45-48, 50-52
Law of double complement, 74 Series, 54
Law of double negation, 23 Set braces, 70
Laws, 23 Set theory, 69-80
Logic forms, 9 1 1, 14, 16-18,29,51 Sets, 69
Logical equivalence, 21-22.24-26. 51 empty, 71
Logical operations, basic, 1, 15.40-42 power, 72
Logical symbolism. 1.9, 16-17 Sheffer stroke, 42
Simplification, 58-60
Subsets, 69
Members, 69
Substitutive instance, I5
Switches, 53
Negation, I Switching circuits, 53-67
Normal forms: Syllogisms, hypothetical, 50
conjunctive, 29-35 Symbolism, logical, 1,9, 16- 17
disjunctive, 35-40 Symbols, variable, 9-1 1, 14, 16,29
Not, 1
Tables, truth, 10-1 1, 16-18
Operations, basic logical, 1, 15, 40-42 Tautologies, 14-15, 33.45, 51
Or, 1 Terminals, 53
Theorems (Boolean), 87-91
Transitivity rule, 24-25
Parallel (switches), 54
Translations, alternative, 7-8
Parentheses. 9
Truth tables, 10-1 1, 16-18
Postulates (Boolean), 85
Truth values, 2-6.8, 14
Power sets. 72
Remises, 45.5 1 , 7 1
Union, 71
Principle of duality, 87 Universes, 70
Proper subsets, 69
Validity, sententid, 45-48, 50-52
Quantification theory, 48 Values, truth, 2-6.8, 14
Variable symbols, 9-1 1, 14, 16, 29
Replacement rule, 24 Venn diagrams, 72
TESTSIMATHEMATICS LEVEL I
James J. Riuuto
MATHEMATICS WORKBOOK In 307 sequential review sec- HOW TO PREPARE FOR COL.
FOR COLLEGE ENTRANCE EX- tions, this self-diagnostic guide LEGE BOARD ACHIEVEMENT
AMINATIONS - - covers such essentials as e l e TESTS1MATHEMATICS
Provides intensive practice in mentary algebra, plane geome- LEVEL l l
arithmetic, algebra, geometry. try, binomial theorem, coordin- Howard Dodge
number series, roots and radi- ate geometry, quadratic equa- A practice and review guide for
cals, data interpretation, data tions, exponentsand logarithms, t h i s more advanced College
sufficiency, and all the other permutations and combinations, Board Achievement Test. With
topics covered on the SAT and and trigonometry. With 7 sample practice questions covering
other mathematics aptitude examinations; a l l answers functions, trigonometry, prob-
tests. Contains 185 drill exer- explained. $4.25 ability, logic,conic sections,and
cises, 662 word problems, 20 other topics tested on the exam,
complete aptitude tests. All plus practice tests with answers
answers fully explained. 54.95 explained. 54.25

HOW TO PREPARE FOR AD-


VANCED PLACEMENT EXAM.
INATIONS I N MATHEMATICS
COLLEGE ALGEBRA Shirley 0. Hockett METRICS MADE EASY
Max Peters Coversall the topics listed by the Fully illustrated guide contain-
T h i s comprehensive text ex- College Entrance Examination i n g many u s e f u l c o n v e r s i o n
plores every topic of college or Board for both "Calculus AB" tables and charts. Givesvaluable
advanced high school algebra (elementary calculus) and "Cal- hints on how t o learn metrics
courses. The language is simple culus BC" (through differential logically and accurately.
and direct; illustrative examples equations). Four multiple choice Features a visual comparison of
with detailed explanations make examinations provide practice m e t r i c and customary equiv.
each solution clear step-by-step. for the first part of the AP exam alents and explanation of metric
The e x e r c i s e s are c a r e f u l l y and 15 sample essay exarnina- m a t h with sample problems
related to theory and are of grad- tions prepare students for the worked out in detail. 5.75
eddifficulty. 54.95 second part. All answers fully
explained. $6.50

At your bookseller or order direct addmg 10% postage plus applicable sales tax.
BARRON'S EDUCATIONAL SERIES. INC.. 113 Crossways Park Drive. Woodbury. NY 11797

You might also like